Consulting Cases

Pataasin ang iyong marka sa homework at exams ngayon gamit ang Quizwiz!

Case Interview Question #00359: The President & CEO of Comcast commissions you to look into a concern that he has. He has heard that DirecTV is planning to launch a commercial satellite that will offer satellite TV services to the domestic market. Is this is a credible threat to Comcast's monopoly position in cable comcast cable tvTV services? If it is credible, what should Comcast do?

Additional Information: Comcast (NASDAQ: CMCSA and NASDAQ: CMCSK) is currently the largest cable operator, home Internet service provider, and fourth largest home telephone service provider in the United States. Headquartered in Philadelphia, Pennsylvania, the company provides cable television, broadband Internet, and telephone service to both residential and commercial customers in 39 states and the District of Columbia. DirecTV (NASDAQ: DTV) is an American direct broadcast satellite service provider and broadcaster based in El Segundo, California. Its satellite service transmits digital satellite television and audio to households in the United States. Possible Solution: Suggested framework: Profits = Revenues - Costs = Price x Quantity - (Fixed Costs + Variable Costs) Upon inquiry I find that it would cost DirecTV $5 Billion to get this satellite in the air and operational. I also find out that DirecTV will offer 200 channels whereas Comcast only offers 50 channels. At first, I had a tough time putting a structure to this business competition type of case. In the end, after asking a few questions, I determined that we needed to estimate the revenues and costs for DirecTV to see if this was a credible threat to Comcast or not. 1. Price I assumed that on average, current cable bills are $35/month. In order to be a credible threat DirecTV needs to be able to provide service at a price of around $35 (could be a little higher because of additional channel offerings). 2. Quantity On the quantity side I estimated the number of customers that the satellite TV offering could reach. I start with 100M households in the U.S., estimate that 60% of those are in the market for cable services (combination of income levels and need for cable service to get any reception, which differs significantly by region). I find out that Comcast has 40M customers across the country. I assume that all of these households could potentially be in the market for satellite service if it was priced right. 3. Initial Costs and Amortization The initial $5B cost needs to be amortized over the useful life of the satellite. Upon inquiry, I discovered that in general a satellite could last for 7-10 years. However, due to the rapid evolution of technology, the actual useful life of 1 particular satellite would be 5 years. This means that $1B needs to be amortized each year over the customer base of the satellite TV services. 4. Fixed Costs I estimate that DirecTV may be able to penetrate 50% of Comcast's existing market (40M * 50% = 20M), then the fixed costs would be ($1B / 20M = $50 per year). In addition to this cost, DirecTV has to cover the costs of sales and service as well as attaining rights to cable programming. 5. Variable Costs An additional cost that needs to be amortized is the cost of the satellite dish for the consumer. It's useful to mention that DirecTV could also opt to lease dishes to consumers to get around this big up front cost for the consumer. I discovered that the combined operating costs would, if DirecTV chooses the leasing of the dish method (like today's consumers lease the cable box, and it gets rolled into the monthly bill of $35), be low enough to make this a credible threat to Comcast! Recommendations for Client: The threat from DirecTV is credible! Some suggestions that I had for Comcast to address this were: 1. Attempt to provide their own satellite TV service. 2. Buy out the DirecTV offering. 3. Improve (increase) the program offerings to your current customers to close the gap on what the satellite would offer. 4. Try to develop exclusive agreements with certain TV program providers to lock out the satellite TV offering.

Case Interview Question #00310: OPERATIONS, R&D PRIORITY, ELASTICITY OF DEMAND (PAY MORE IF IT'S IMPORATNT), IMPLEMENTATION (R&D, MASS PRODUCTION, EASE OF SELLING, CONJOINT ANALYSIS) Your client is the president of GM Research Lab, which is essentially an electronics Research & Development (R&D) think tank for General Motors Company (NYSE: GM). Headquartered in Warren, Michigan, the GM Research Lab is a very innovative place which employs about 100 scientists GM onstar systemand engineers who buy components and create prototype innovative products for cars. This place is bursting with energy, and they already have many great products to their credit. For instance, they have already developed a prototype car autopilot navigation system. Similarly, they discovered that many of the road deaths due to car accidents are preventable. The problem is that medical services don't reach there in time either because they don't get to know about the accident at all or they don't know exactly where it happened. In most such accidents, the victims either lose consciousness or are decapacitated. So they have created this device which is a combination of GPS, airbags and cell phones. The way it works is that on car accident impact, the airbags inflate, and the cell phone calls 911, pinpointing the directions using the GPS (think of GM OnStar, Ford Sync, BMW Assist, Toyota/Lexus Safety Connect, etc). These guys are full of ideas, and some of the projects under consideration are night-vision for cars (using F-14 technology), backing sensors and so on. However, their costs range is high. The fundamental issue is that the engineers are like kids in a candy shop. They are doing very exciting work and love their jobs. However, there are random projects going on all over the place. The president wants to streamline the operations and focus on 10-20 projects rather than the more than 100 that are going on right now. The president has asked you to help him address the following questions: How will you prioritize the projects, deciding which ones to focus on and which ones to abandon? How will you convince the engineers, who are basically innovative brains and do not understand business, of your decision?

Interviewer: This is a case on which I worked on about 8-10 years ago. I like this because it is different from most usual problems. How would you go about it? Candidate: Interesting case. So, to prioritize the projects, I will probably start by the basic economics, focusing on the projects that are likely to bring in the most profits. Thus, some of the factors I will look at are: Probability of successful completion of the projects Probability of an automobile manufacturer buying the technology for commercialization Demand and price elasticity in the market Speed to market because money has a time value Practicality of the idea in general. To convince the engineers of the decision, I would focus on the following issues: Speed to market Ease of selling Practicality of mass-production Share of royalty Profits: Success of the company, and individual success Interviewer: Okay, interesting. Let's take these one by one. You said that you want to prioritize based on the economics. How do you get a handle on the expected profits? Candidate: I will proceed in a structured manner, starting off with Profits = Revenues - Costs (or expected profits = expected revenues - expected costs). We can find these figures by subjectively assessing probabilities of success in different durations and to different levels and calculating the expected monetary values. I am assuming that the costs aspect could be as follows: I would probably not want to break this down into fixed and variable costs as you said the company is engaged in creating prototypes. Thus, variable costs are wiped off the slate since quantity produced is equal to one. They only have R&D costs which can be split into human resources, material and overheads apportioned. The human resources costs would likely follow a similar pattern, and would depend on the time taken in project completion. To deal with the revenues, I will use the simple equation: Revenues = Price x Volumes. product life cyclePlease understand that since we are dealing with prioritization, I am looking at revenues, costs and profits for individual projects rather than the entire company. I guess we can have some estimate of the volume by looking at the product life cycle (see Figure 1). At prodcut launch we would have only the innovators and the early adopters and we'll have to cross the chasm before being able to reach an early majority. Normally, I would think, the only role costs play in pricing is to make sure that at the volume and pricing, costs including R&D and fixed+variable costs of mass production are covered. The pricing is typically based on what the consumer is willing to pay for it. To be able to make a better analysis of the revenues, both from the prices perspective and the volume perspective, I will need to understand how this firm earns its revenues. Can you help me with that? Interviewer: Sure. The company has good relations with an automobile manufacturing company to which it sells its ideas. The automobile company hands it a royalty check based on estimated revenues depending on how they can price it. They also have the option of earning the royalties over 5-6 years based on actual revenues. Candidate: So that rules out cost-considerations, pretty much? Interviewer: It does. Candidate: Is it fair to assume that volumes lose much of its relevance as the automobile company will fit the product in all cars of a particular model. Interviewer: That is a fair assessment. Candidate: Since pricing for end-consumer seems to be an important factor here, I'd want to explore what the end-consumer is willing to pay for the product. Interviewer: And how would you go about doing that? Candidate: By conducting some market research. Maybe even conducting a conjoint analysis on automobiles with different products, and prioritizing on the basis of that? Interviewer: What is conjoint analysis? Candidate: Conjoint analysis is a market research technique wherein the customers are presented with products with different attribute options, and their responses are analyzed to figure out which attribute options they prefer and how much they value each. Ummm...but yeah, there's a problem that to conduct this analysis we need prototypes, and here we want to do the analysis before the prototype is created. I guess that could be solved by paper prototyping or giving out the product idea in some way. Interviewer: Hmmm....but this method assumes that consumers know about their preferences. The engineers would argue that since these are very innovative products, consumers don't have enough information to develop sensible preferences before they can actually use them. Candidate: Perhaps we could try using an economic-value based pricing model then. Interviewer: What does that mean? Candidate: It essentially means we find out how much economic benefit the consumer gains from using the product. That economic benefit is then split between the consumer and the manufacturer. Interviewer: Okay, so how do you gauge the economic benefit a GPS has over a $5 map? Candidate: I guess we could look at the average time saved, put it against the average worth of the consumers' time. Ummm....I guess we are not reaching anywhere with this approach. Could we possibly compare the products to similar products produced by competition? Interviewer: For normal products you can do reference pricing. But, as I said, these are highly innovative, completely new products. There is nothing you can compare any of these new products to. Candidate: I guess the only way left is to look at the products sold in the past or selling at present. I suppose it should be possible to determine price-elasticities of demand for different categories of products. For instance, demand would normally be less elastic for safety products. In other words, people would be willing to pay more for products that they perceive as making them safer on the road, just like they are normally willing to pay more for kids' pharmaceuticals. product vaule vs frequency of useInterviewer: Right! That's exactly what we did. We are almost out of time here, but let me tell you what we did in this case. Following the same logic as you did, we concluded that there are different attitudes that people have towards different types of products. We plotted all their products on two axes - value and frequency of use. We found that there was a pattern like this (Figure 2): The products that are infrequently used but are perceived as having high value are safety products - in fact you wish you never have to use them. The GPS-Airbag-Cellphone combination I mentioned earlier falls in this category. Then there are products that are useful, but are used relatively frequently. These are convenience products like GPS. Then there are items that are less useful but are very frequently used....like power windows. These, we called utility items. Invariably, products in the top-left corner were more successful. That's where we took the leap of faith, and said that in general people are willing to pay more for safety products, and are more likely to buy them. We recommended that the client company prioritize their products on the basis of this chart...based on where the proposed product falls on this chart. Which, of course, they did. Candidate: Wow! This was indeed a very different and interesting case. I thoroughly enjoyed it. Interviewee's Note: This business strategy case mainly involves process optimization and prioritization. It was an extraordinary case in several ways. For one, the interviewer spoke more than I did, which is very uncommon for a case interview. Then, the way the case is set up, applying traditional frameworks to crack it takes up a lot of time, though at the end you do reach close to the answer as I did. Lastly, this is the only McKinsey case without numbers that I have seen.

M&A, HOW IT AFFECTS BETA? The client Fomento Económico Mexicano, S.A.B. de C.V., doing business as FEMSA (BMV: FEMSA, NYSE: FMX), is the largest beverage company in Mexico and in Latin America. Headquartered in Monterrey, Mexico, FEMSA is also the largest independent Coca Cola FEMSA beerbottler in the world as of September 2011. For this case, FEMSA is interested in selling its beer business to Heinekin or SAB Miller. FEMSA is based mostly in Mexico. Should FEMSA go ahead to sell its beer business? Why or why not? Don't focus on the price of the transaction, but focus on the strategic view of the case.

Additional Information: (Provide the following information if requested by the interviewee.) FEMSA is consumer goods centric with three major business units: The first division is Beer with $3 B in annual revenue. Its beer division has 7 - 8 brands. 75% of the total revenue is produced in Mexico and the other 25% is in US. They have famous brands such as Dos Equis, Tecate, and Sol. They've seen 3% - 5% growth, which is in line with their mature market. Their biggest competitor is Modelo who has been more profitable. Modelo (manufacturer of Corona beer) has a 55% market share and FEMSA has 45%. 50% of Modelo's revenues comes from Corona. FEMSA has 20% operating margins. Although this market is a duopoly, the two players FEMSA and Modelo have tried to avoid price wars, so they are cooperating to a degree, but not a ton. The second division is CocaCola FEMSA. This is the bottling facility for Coca Cola. It's the largest in Latin America and the second largest in the world with $7 B in annual revenue. This division is the fastest growing business within FEMSA. CocaCola FEMSA has made lots of acquisitions and has had strong organic growth. Needless to say, this division is highly profitable. The third business is FEMSA Retail ($4 B in revenue), which consists of a chain of convenience stores similar to 7-11. FEMSA Retail is the most dominate player in the market, and it has very fast growth, ~30% a year. As it typical of the retail world, FEMSA Retail's margins are slim. They've managed to obtain a 6% margin, which is above industry average. In the last 10 - 15 years, there have been lots of mergers and acquisitions in the beer industry. The three major players in the beer industry are SAB Miller, Imbev, and Heinekin. Local brands tend to dominate only in local markets. There is strong emotional aspect to local brands, but most brands are moving toward a global reach. Typically this is because there's sourcing, manufacturing, and distribution synergies that help. At this point you are free to choose the side of FEMSA or Heinekin. Let me know what you choose and what you would advise the client. Possible Answer: I chose FEMSA, based only on the fact that the interviewer had tons of information regarding FEMSA. I asked whether FEMSA is public or private. It turns out FEMSA is publicly traded. That changes the equation and how the CEO can approach this case. Five buckets that will have an impact on share price: competition strategic operational investment options diversification premium Within Competition, I talked about Modelo being a faster growing company, being squeezed out of industry, and new entrants coming in. Within Strategy, I talked about duopoloy currently in place and possible expansion plans. Within Operational, there is certainly more value in a large brand. Any synergies that could be sold. Any profit sharing or results sharing opportunities. Investment options - When the money does come in, what to do with that money. Dividends, another acquisition, grow current businesses, improve current operations. Diversification premium: when you're in 3 different business, you're dinged in share price. The value of the firm is worth more separately. Transaction itself: the CEO needs to know what the right selling price is. Don't get into numbers, but talk about it. Transaction price. How the transaction would change the industry's beta and company's beta. Once you get out of the beer business, FEMSA is no longer in the same industry as a company. FEMSA was originally a consumer goods company. Once the beer business is sold, it's more a distribution company. The profile of the company changes. There will be a change in shareholder. This will also change the company's beta. As a CEO, you will need to understand these issues. Institutions that would previously fit in their beverage portfolio would no longer fit. FEMSA is facing the risk of getting squeezed out of industry because we're growing slower than our competitor. Given the trends, we will be squeezed out. If a third competitor comes into the equation, competition becomes harder to maintain and the company could be squeezed out further. Based on all these considerations the final recommendation to FEMSA is to sell the beer division.

Case Interview Question #00621: PROFIT, DIFFERENT COUNTRY Ghana, officially the Republic of Ghana, is a country in West Africa with a population of about 24 million (as of 2010). The capital and largest city is Accra, with an estimated urban population of 2.3 million as of 2012. Ghana is ranked as a Lower-Middle Income Economy by the World Bank. In 2011, busy internet cafe in ghana27% of Ghana's population were living on less than $1.25 per day. Ghana's capital city of Accra boasts about 500 internet cafes. Our client is Busy Internet, the largest internet cafe in Accra and in Ghana. Recently, Busy Internet is experiencing a decline in profit. The Managing Director of Busy Internet, Ms. Estelle Akofio-Sowah, has hired your consulting firm to asses the problems. In addition to identifying the cause of declining profits, you also need to give recommendations and assist them through the implementation process. How would you about this case?

Additional Information: (to be given to candidate if requested) Besides the regular product/service provided by an internet cafe (like emails, web surfing, video games), Busy Internet also offers a range of other related services: a business incubator service ISP, Internet Service Provider (broadband, dial-up, wireless, satellite, and radio connection) Copy Center, equipped with printers, copiers, scanners, etc. Conference Room Office Renting Suggested Approach: The purpose of this case is to asses the candidates' analytical skills and his/her abilities of gathering information through appropriate questions. A good understanding of the Ghanaian business climate is important but not a must. A good structure to answering the case is necessary. The interviewee should suggest a possible framework and a solution but he/she should have a clue about the major problems of implementing changes in a non-traditional business environment like Ghana. Gathering information in Ghana is extremely difficult, and managers tend to flood consultants with information. Therefore a good structure, the ability to prioritize and navigate through useless or cloudy/ambiguous information is important. For these reasons, the interviewer can give upfront useless information and hide the important ones. Since Ghana best practices are still to be polished, it is important that the interviewee display a certain degree of flexibility and adaptability while asking questions. The use of inflated and difficult terms should also be avoided. Possible Answer: Interviewer: So how would you analyze this case? Candidate: It really looks like an interesting and complex case. May I have a second to structure my analysis? Interviewer: Sure. Candidate: OK, so these are the basic steps of my structure. First, I want to know more about the products. It looks like Busy Internet has a very diverse product/service mix. So I want to know more about the service Busy Internet is selling to its customers, they can have different profit margins. Next, since Profit = Revenues - Costs, I want to analyze these 2 elements separately. Finally, I want to analyze the market's trend for each of these products/service. Interviewer: Sounds great, let's start with the product mix. Busy Internet sells a big variety of different services to its clients, but we can group the major sources of revenues in 3 different products: Copy Center Cyber Cafe ISP, Internet Service Provider Candidate: OK, let's analyze these 3 divisions separately. Interviewer: All right, what's next? Candidate: What do we know about the costs for these 3 divisions? Have they increased lately? Interviewer: Ghana experiences black outs, and in order to maintain the service Busy Internet must use a very expensive power generator. Apart from that, the 3 product lines have not experienced any increase in cost. Candidate: What do we know about the revenues for these 3 divisions? Interviewer: The copy center is experiencing a decrease in revenues; the same can be said for the cyber cafe while the ISP is slowly growing. Candidate: That is interesting. Do we know any information about the global markets for all these products? Are these markets growing in Ghana? Are they fragmented? Interviewer: OK, here is some more information: Copy center: it's a growing market, many new small players are entering the market making lots of money Cyber Cafe: internet cafes are not doing very well in Ghana. Many internet cafes are closing as more Ghanaians are enjoying internet connection at home ISP: it's a growing market, technology is improving and it is now possible to reach many customers via radio and wireless Candidate: Thank you. What do we know about the competitors? Interviewer: Good question. There are new entrants in the Cyber Cafe market, a new internet cafe called Mega Internet is stealing customers from Busy Internet Cafe. Competitors in the ISP market are mostly minor players with the addition of Ghana Telecom, the government sponsored Internet Service Provider. Candidate: What is our value proposition to our customers? Similarly, how is the competition targeting our customers? Interviewer: Busy Internet customers are elite customers, prices at Busy Internet are very expensive (the highest in Accra), and cusomters expect high quality. Not many competitors can target the same segment of customers. We can say that there is only one big competitor that can offer similar quality compared to Busy Internet cafe. Candidate: The scenario now looks clearer. Our revenues for the cyber cafe are declining for a decrease in volume and not in price. Why is Busy Internet loosing clients in the cyber cafe? Interviewer: Good questions. How would you look for that? Candidate: My idea would be to ask the customers. Also, we can conduct quality survey, supervise customer behaviors, and assess employees' performances. Interviewer: It looks like Busy's customers are gradually moving to another cafe Mega Internet since customer care and service quality is better. Also PCs in the internet cafe at Busy are getting old and many of them are not working properly, decreasing actual capacity and customer loyalty. Candidate: I have another question, I am still not sure why our revenues in the copy center are decreasing while the market and demand for copy center is growing. Is that connected with the decrease in traffic of the internet cafe? Interviewer: It could be. What other possibilities would you investigate? Candidate: Well, there are many possible issues Quality: our printers or copy machines are not competitive Customer service: our workforce is not trained Efficiency: probably our printers and copy machines are getting old and they break more frequently, decreasing volume and customer loyalty Capacity: Busy Internet has probably not set up the right sufficient capacity to meet the growing demand for copy center services Interviewer: Good points, they all make sense. Ms. Estelle Akofio-Sowah, the managing director of Busy Internet, has just reached us to ask for the executive summary of your analysis. What should we tell her? Candidate: It looks like the decrease in profit is due primarily to decrease in revenues and in volumes. Revenues Cost Market Busy Internet's Market Position Copy Center declining stable growing average player Cyber Cafe declining stable declining major player ISP growing stable growing major player Busy Internet must concentrate its investment in the copy center, since it is a growing market. The cyber cafe is a declining business in a shrinking market, but Busy Internet should keep the cyber cafe and use it to cross sell other products like ISP and copy center services. Busy should invest heavily in customer care, employee training, performance incentives, technology improvement in order to perk up services and increase cyber cafe customers' loyalty. ISP is growing. Busy Internet should leverage its customer base, client portfolio and its economy of scale to anticipate and satisfy customer expectations. Optional Bonus Questions on implementation Interviewer: Ms. Estelle Akofio-Sowah looks impressed of your analysis. But she also looks skeptical on some of the points you have just mentioned. She thinks that there are no major problems on the technology side as well as the customer care. Investments on these areas are expensive and additional funds from shareholders have to be motivated. How would you try to convince her that these changes are vital? What approach do you think could convince her? What about the other managers involved in these change? What do you expect will be the major issues in implementing change at BusyInternet? How can these changes be sustainable and long-lasting? Why do you think Ghanaians resist changes and like their status quo? These questions are like "bonus questions" and are meant to be asked only to these candidates that developed a very good understanding of the economic/problem solving side of the case and are ready to give suggestions also on the implementation/leadership side. Rather than asking about previous leadership experiences in which the candidates demonstrated motivation, persuasion and other interpersonal skills, this case offers the opportunity to "bring the interviewee to Ghana" for 20 minutes and assess his/her economic and interpersonal skills on a real African situation. The last part of the case can be easily changed on a role-play interview so to give a more realistic taste to the case discussion.

Case Interview Questions #00067: The CEO of Nintendo Co. Ltd., a large diversified entertainment corporation has asked a McKinsey team to examine the operations of a subsidiary of his company that manufactures video games. Specifically, he needs to know if he should approve a $200 million capital request for tripling nintendo video gamesthe video game division's capacity. You are a member of the McKinsey team assigned to this project. Assume you and I are at the first team meeting. What are the critical issues we should plan to examine to determine if the video game industry is an attractive one for continued investment and why?

Additional Information: (to be provided if asked) The following information may be given if requested by the candidates though you should focus on having the candidate identify issues, not obtain more information. Market Share Nintendo's video game division is the third largest manufacturer of hardware in the video game industry with 10 percent market share. Top two producers have 40 and 35 percent market share (think about Nintendo Wii competing with Microsoft's Xbox and Sony's PlayStation). Remainder is divided by small producers. Division sells to broad range of consumers. Sales Division sales have increased rapidly over last year from a relatively small base. Current estimate is annual sales of 500,000 units. Current estimate of industry hardware sales is 5,000,000 units annually. Industry growth has been strong though over last few months, sales growth has slowed. Divisions current sales price for the basic unit is $45 per unit. Division remains less than 20 percent of parent company sales. Top two competitors also develop, manufacture and sell software/games though division sells only licensed, software. Industry growth of software continues to increase. Costs Division estimates current cost is $30 fully loaded. Requested expansion should reduce the cost by 5 to 7 percent and triple production of the hardware units. Top two computers are estimated to have a 10 to 15 percent cost advantage currently. Main costs are assembly components and labor. Customers Division estimates much of initial target market (young families) has now purchased the video game hardware Nintendo Wii. No large new user segments have been identified. Distribution Primarily outlets of distribution are top end electronics stores. Profitability Division currently exceeds corporate return requirements; however, margins have recently been falling. Product Hardware standards have been established by the industry leaders. Product features constantly developed (e.g., new remote joy stick), to appeal to market segments. Note to the Interviewer The main purpose of the case is to determine whether the candidate is able to structure a basic industry analysis. The primary issue of the case is to determine if the industry is attractive and, especially, if our client's position in that industry is sustainable. The candidate should identify issues which are necessary for assessing both the industry and our client's position, but should not be expected to solve the problem. If the candidate begins to discuss too deeply a specific issue, before having covered the key issues overall, bring them back to discuss the industry more broadly by asking "what other issues must be examined?" If the candidate is discussing issues which seem irrelevant to the attractiveness of the industry, ask "how will that analysis help to assess the attractiveness of the industry or our client's position". Then, ask the candidate to identify other issues which must be examined. Possible Solution: The following issues would need to be covered for the candidate to have done an acceptable job: 1. What is future market potential? Candidate needs to question the continuation of overall industry growth. She/he might ask about the saturation of markets, competitive products (home computers), and declining "per capita" usage. 2. What is the competitive outlook? Candidate should at least recognize the need to examine competitive dynamics. Issue areas might included: concentration of market shares; control of retail channels; and R&D capabilities (rate of new product introductions, etc.). 3. What will be the price/volume relationship in the future? Issues of prices need to be considered. Better/Outstanding Answer: No bounds on creativity, but better answers would address the following: 1. Market Potential Recognize that there is a relationship between market penetration and growth in new users which, when combined, yields an industry volume estimate. Address the shifting mix of product purchases, in this case from hardware (video game consoles) to software (game CDs/DVDs). Seek to look at buyer behavior in key buyer segments, i.e., "fad" potential of product. 2. Software Recognize technology standards are set by industry leaders. In this situation, the division as a secondary player will have to follow these standards. Recognize that different distribution needs may exist for different products (In this case, hardware versus software). Discuss the effect capacity additions can have on overall industry price/volume relationships and on industry price levels. 3. Company's Ability to Compete Should ask what the capacity expansion is designed to do. Explore the cost position of the client division relative to that of other competitors. Seek to understand reason for poor profit performance of division.

Case Interview Questions #00366: NOT GIVE RECOMMENDATION, GATHER DATA, LOOK AT PORTERS 5 FORCES, The client BASF SE (FWB: BAS) is a large global chemical company headquartered in Ludwigshafen, Germany. BASF has retained McKinsey to evaluate Monheim, Germany-based Cognis, another large player in the chemicals industry, as a possible merger/acquisition candidate. Both BASF and Cognis basf cognis acquisitionare bulk commodity chemical producers. Our consulting team's main job is to analyze the future prospects of the target company's main product line: a bulk chemical that is used in the production of plastics. Our consulting team has to analyze Cognis, the target company's future prospects in its major product line. As the team leader of this case assignment, what information do you need to know and how would you structure the analysis to determine if the proposed merger/acquisition is a good idea or not?

Additional Information: (to be revealed during the course of the case interview) Production of this chemical has been declining slowly for several years. Prices have declined rapidly. There are 8 major producers of this chemical with market share divided the following way: the largest producer has a 30% share, the second largest has a 20% share, and our target company has a 15% share. The rest is divided among the remaining 5 competitors. Profitability in the industry is relatively low. The two largest competitors earn small returns, our target company is breaking even, and the remaining competitors are operating at a small loss. The largest competitor has just announced plans to substantially increase capacity. Possible Solutions: This is a merger/acquisition and industry analysis case in which Porter's Five Forces framework might be useful, but it is really an exercise in how to uncover the underlying drivers of the facts given in the case. In general, the interviewer was more interested to see if I could ask the right questions to collect data. There are countless paths that the analysis could flow down. Presented here are some of the most obvious to consider. A satisfactory response would include addressing the following issues: 1. Industry & Market Analysis Candidate: What buyers or target markets make final use of this chemical product? What is the nature of the growth and profitability of these end markets? Interviewer: The chemical was used primarily in the automotive related industries. Candidate: Is this industry or product regulated in any way that affects cost, pricing, or profitability? Interviewer: Environmental and pollution regulation apply in the normal way to the chemical production process, but nothing out of the ordinary. Candidate: How much production capacity exists in the industry compared to the demand today? And compared to the estimated demand in the future? Interviewer: Much more capacity than necessary. Candidate: What is the current capacity utilization in the industry? How about our target firm Cognis? Interviewer: Both the industry and our target firm are operating at approximately 70% capacity. 2. Company/Product Candidate: What is the relative cost position of our target company Cognis compared to the rest of the competitors? Interviewer: Our target company has a reasonably good position compared to the rest of the industry in terms of size, age and efficiency of equipment, and financing. Candidate: How diversified is this target company and does this product represent a significant source of revenue? Interviewer: It is a significant source of revenue, but the most competitive producers are adequately diversified. Candidate: Are there additional niche or value-added uses for this chemical or its by-products that are as-yet untapped? Interviewer: Nothing really significant. 3. Competitions Candidate: How rational or volatile is pricing between competing firms? Interviewer: The industry players often engage in price cuts to temporarily increase market share, but usually suffer falling profitability as a result. Candidate: Do we know the reason for the largest competitor announcing capacity increases? Are they trying to introduce a credible threat to deter future entry or expedite exit from the industry? Interviewer: We are not sure but it could be possible. Candidate: Are entry or exit costs prohibitive? Interviewer: Market entry is expensive due to the unique fixed costs of producing this chemical. Exit is relatively inexpensive. Candidate: Has the number of competitors or their market share changed significantly in recent years? Interviewer: Competitors have been in this industry for a long time and many plants are fully depreciated, making exit inexpensive. Market share has not changed significantly in recent years. 4. Synergies in Merger/Acquisition Candidate: Are there operational improvements that the target company Cognis could make to enable it to be more efficient or other management expertise that our client's company could bring into the merger/acquisition? Interviewer: Yes, on both accounts. Candidate: Are there scale economies in production or distribution? Interviewer: Yes, economies of scale exist in marketing and transport, but are much smaller in production. Candidate: Are there other synergies between our client's company and the target company such as product mix, cross-selling opportunities, raw material purchase, etc? Interviewer: Not significant at this point.

Case Interview Question #00426: COMPETITION RESPONSE, HOW TO LOWER FIXED COSTS, HUMAN CAPITAL QUESTIONS WITH UNION SALARIES Our client is Deutsche Post AG (FWB: DPW), one of the world's largest logistics groups headquartered in Bonn, Germany. Deutsche Post is the successor to the German mail authority Deutsche Bundespost, which was privatized in 1995. The Mail division of Deutsche Post inherits most of the traditional deutsche post DHLmail services formerly offered by the state-owned monopoly, for which it uses the Deutsche Post brand. Historically, they have been the sole provider of mail delivery service in Germany. They have exclusive right to deliver letters under 50 grams in Germany. A few months ago, however, following the implementation of European legislation the German government passed a bill to deregulate the German mail delivery industry. The client is forecasting a 20% drop in revenue, and they would like to know how to proceed. What would you recommend Deutsche Post to do?

Candidate: I just want to make sure that I understand the problem correctly. Our client, Deutsche Post, is facing deregulation in its industry. They are anticipating a 20% drop in revenue, and would like our advice on what they should do. Interviewer: Correct. Candidate: I see. Before I structure my analysis, I would like to ask a few clarifying questions. What does this deregulation mean? Interviewer: Traditionally, any company other than Deutsche Post was legislated by law to charge at least 4 times the price of Deutsche Post for delivering mail within Germany. This means that if Deutsche Post charged $0.25 per letter, then no competitor is allowed to price their service below $1.00 per letter. This regulation has now been abolished. Candidate: I think I understand. So an insurmountable barrier to entry has now been removed, and the 20% anticipated drop in revenue is going to come as a result of competitors entering the industry. Interviewer: Yes. Candidate: Great. May I have some time to structure my thoughts? (Note: I would encourage you to do the same, and get some practice with the upfront structuring of cases.) Interviewer: Of course. Candidate: I think I'm ready to begin. In order to understand the problem, I would like to start off by quantifying what the 20% drop in revenue will mean. To do this, I would like to examine the revenues and costs of Deutsche Post first. Next, I would like to investigate four key areas to see how our client can respond. To start off, I want to examine the Competition. I want to understand who they are, and how they are able to take business away from us. Next, I would like to look at our Customers. I want to have a good understanding of who they are, what they look for, and the different segments that are present. Third, I want to take a look at our Product offerings, and see if they are in line with what the customers want. In addition, I also would like to see how our offerings compare to our competitors. Finally, I want to investigate the Company itself, and understand its core competencies, financial situation, and other factors such as organizational structure to see what kind of a response our client is able to conjure. Interviewee's Note: The structure of my analysis was basically broken up into two major sections. The first section includes revenues and costs. This section is dedicated to figuring out the quantifiable effects of the revenue drop, and "What is happening" from a quantitative standpoint. The second section includes customers, competition, company, and products. This section is dedicated to figuring out "What to do about declining revenues", as well as gathering qualitative information. Interviewer: Sounds like a reasonable plan. What would you like to know first? Candidate: To start off, I want to quantify the effect of this 20% drop in revenue. Before I dive in, is it our job to verify this revenue forecast? Interviewer: You can assume that it's accurate. Candidate: Excellent. May I ask how much revenue our client currently generates? Interviewer: One billion dollars. Candidate: I would like to take a closer look at the revenue, and break it down into its components. Do we have any information on that? Interviewer: What do you think are the components? Candidate: There are lots of ways to segment it, but I know that revenue is comprised of the average price per unit times the volume of units sold. Interviewer: That's fine. So let's say that on average, the price per delivery is $1.00 Candidate: So if our revenues are 1 billion dollars, and the price per deliver is one dollar, then our volume must be around a billion deliveries last year. However averages can be deceiving. Is there a big price variation between certain types of deliveries? Interviewer: Yes, but for this case you can assume all deliveries are made at one dollar. Candidate: You mentioned that our client is anticipating a 20% drop in revenues, and I am interested in know whether this will be due to reduced volumes, or reduced pricing. Do we have any information on that? Interviewer: They are expecting reduced volumes. Candidate: Interesting. So it seems like the new entrants to the market will most likely be taking business away from us by reducing our volumes. I think I understand enough regarding the revenues. Their revenues will drop from a billion dollars to 800 million dollars due to a drop in volume that is caused by new competitors. I would now like to move over to the cost side of the analysis, and see what's going on there. Interviewer: Sure, what would you like to know? Candidate: In the same time period where we generated a billion in revenue, how much were our costs? Interviewer: 800 million. My response: Do we have a further breakdown of costs? Interviewer: How would you like to break them down? Candidate: Let's go with fixed and variable. Interviewer: Sure. The fixed costs are 700 million, and the variable costs are 100 million. Candidate: Wow, that's a high fixed cost. Am I right in assuming that the fixed cost will not change at all when our revenue drop 20%? If this is the case, and variable costs drop proportionally with revenues, then it means that our profits will go from 200 million to 20 million - a drop of 90%. Interviewer: Your math is correct. Candidate: I think I understand everything I need to know about revenues and costs. Our numbers before the deregulation was a billion in revenues, and 800 million in costs. However, due to the extraordinarily high fixed cost, the 20% drop in revenue will actually result in a 90% drop in profits. This is not good at all for the client. In order to see what we can do about this, I would like to move on with the upfront structure I laid out, and examine the competition. Interviewer: Sure. Interviewee's Note: Up to this point, I thought that the case was going fairly well. There was a bit of a trap with the high fixed cost. If I didn't segment the costs, and just assumed that they would also drop by 20%, then I would've missed a big point. Candidate: The first thing I would like to understand is how these new competitors are going to take market share away from us. Do we have any information on that? Interviewer: What do you think? Candidate: Since we are an established player, it's probably going to take some sort of incentive for our customers to switch. My guess would be that the competitors are going to come in with lower pricing. Interviewer: You're right. Candidate: That's very interesting. The reason I say this is because our client has an enormous fixed cost, and I thought this would provide a pretty significant barrier to entry for new competitors. Do we have any information on how they are able to have a lower price? Interviewee's Note: At this point, I realized that I probably should have segmented fixed costs further in my previous analysis. This is where the case started to fall apart for me as it took me a very long time to figure out how the new entrants are able to achieve a lower cost. Interviewer: What would you need in order to figure that out? Candidate: I think I would need to know what our costs are made up of, and compare those segments to that of the competition. Interviewer: What do you think the fixed costs are? Walk me through it. Candidate: Intuitively I would say that there are costs associated with collecting the mail from the customers, processing the mail at a facility, and delivering the mail to the recipient. Interviewer: You are correct. What do you think is the largest cost? Candidate: Processing the mail at a facility because this requires a large building with lots of sorting equipment. Interviewer: That's interesting, but incorrect. Let's think about this on a per letter basis. Candidate: Sure. If we're looking at a per letter basis, I would say that the delivery cost would be the highest? This is because the mail sorting facility processes so many letters that the cost for each letter is fairly low. Interviewer: You're right. Candidate: OK great! So do we know if the new entrant's fixed costs are comparable to ours? Interviewer: They are much lower. In particular, their delivery cost is much lower than ours. Candidate: Do we know how they are able to achieve this? Interviewer: What do you think? Interviewee's Note: I spun my wheels on this one for about 15 minutes. It turns out that the new entrants focused on mail delivery to large urban centers. In doing so, the workers can deliver many more units per hour - thus saving the new entrants significant cost with regards to worker salaries. I will now jump to the next portion. Candidate: It seems that in order to compete we need to lower our workers salaries. Has management tried this in the past? Interviewer: The workers are unionized and will not take a salary cut without good reason. Interviewee's Note: I ran out of ideas at this point regarding what to do. It turned out that the key was to renegotiate the service level agreement. Instead of delivering mail six days a week, the new agreement will deliver 4 days a week. At this point we ran out of time. When I practice, we usually keep the cases to a maximum of 40 minutes to somewhat simulate a real interview. Since I missed this key insight, the interviewer gave it to me and asked me to conclude. Candidate: After preliminary analysis, the team has concluded that Deutsche Post should renegotiate the service level agreement with the unionized workers in order to respond to the competitive pressure put forth by the new entrants - which is resulting in a 20% drop in revenue. I believe this to be true for two reasons: 1. The new entrant's competitive advantage is a low fixed cost due to the fact that they only deliver to urban areas where the workers can maximize their productivity. Since we deliver to all of Germany, we must reduce our fixed costs through lower worker salaries. 2. The unionized workers are willing to take a lower salary for a reduced workload. By reducing the working days by 1/3, we will be able to justify the lower salary. Interviewer: Excellent! You did a great job. Interviewee's Note: I believe that I could have cracked this case by doing the following things: Segmenting fixed costs to see what was the largest portion at the beginning. I did this later on, but it would have helped had I done it upfront. Asked some qualitative questions regarding how the company currently operates. This may have uncovered the fact that they currently deliver six days a week. The feedback I got from the interviewer was that this case took some business intuition to solve. The options were basically to increase productivity per mailman or decrease salary per mailman. I feel that the biggest problem with my performance was that it took way too long. This is an actual case that a candidate received in one of the McKinsey Germany offices. Any feedback or general comments would be much appreciated.

Case Interview Questions #00048: Your are the managing director of McKinsey & Company, a large global management consulting firm. Traditional strengths of your firm have been solving strategy and organizational issues. Recently, however, you have noticed an increasing number of your firm's proposals are being rejected because of a lack of consulting firm McKinsey & Companyinformation technology (IT) expertise in your firm. So far, your firm's growth has been strong enough that proposals lost have not hurt annual earnings. Nonetheless, you are becoming increasingly concerned about the need to develop the firm's capabilities in information technology. Question #1: Assuming your concern is valid, what reasons will you provide to other partners about the need to acquire information technology skills? Question #2: Assuming your are able to convince other partners of the importance of IT expertise, what steps would you take to rapidly build IT capacity in this area? Question #3: What are the major risks in executing an IT capacity expansion?

Answer #1: Good answers focus on the value of IT to clients: discussion topics include the increasing importance of information in business, strategic value of information and information flows, importance of information systems for implementing new organizational structures and management control systems. Better answers focus on the costs of losing clients to competitors: discussions included the encroachment costs of having clients talking with competitors about IT problems, risk of losing credibility with clients by not being able to solve a problem. Answer #2: Good answers will focus on various methods to build expertise: buying expertise by acquiring another firm, by raiding IT practices of other firms for a few key consultants, building capacity through recruitment of IT experts and training them to be consultants, building capacity by training current consultants in IT practice skills, establishing a strategic alliance with a IT boutique firm. Candidates should discuss the pros and cons of each method proposed; impact on firm's current culture, cost to the firm, time needed to build expertise, etc. Better answers will realize the importance of stimulating client demand as capacity builds through seminars, articles strategic studies in IT areas... Answer #3: Good answers depend on the expansion methods discussed, but an important issue is the loss of the firm's focus away from just strategy and organization. Better answers will focus on the difficulty of implementation in IT; rapid technological changes in the IT industry require significant ongoing training and development costs; new practice cultures may be significantly different from current culture, especially if "external experts" are brought into the organization.

Case Interview Question #00276: LONG, ROUGH Client's Goal: To determine how to improve its financial situation. Our client is Magna Health, a health care company in the Midwest. It both insures patients and provides health care services. Employers pay a fixed premium to Magna for each of their employees magna healthcare in return for which Magna covers all necessary health services of the employee (ranging from physician care, and medications to hospitalization). Magna currently has 300,000 patients enrolled in its plan. It has 300 salaried physician employees who provide a broad range of services to patients in 6 centers. These physicians represent a wide range of specialty areas, but not all areas. When a patient needs medical treatment in a specialty area not covered by a Magna physician, they are referred outside of the Magna network for care, and Magna pays all referral costs on a fee-for-service basis. Magna does not own any hospitals itself, instead contracting services from several local hospitals. Magna's CEO has retained McKinsey to help determine what is causing the declining profitability and how Magna might fix it.

Case Type: improve profitability. Consulting Firm: McKinsey & Company final round job interview. Industry Coverage: insurance: life & health; healthcare: hospital & medical. Case Interview Question #00276: Client's Goal: To determine how to improve its financial situation. Our client is Magna Health, a health care company in the Midwest. It both insures patients and provides health care services. Employers pay a fixed premium to Magna for each of their employees magna healthcare in return for which Magna covers all necessary health services of the employee (ranging from physician care, and medications to hospitalization). Magna currently has 300,000 patients enrolled in its plan. It has 300 salaried physician employees who provide a broad range of services to patients in 6 centers. These physicians represent a wide range of specialty areas, but not all areas. When a patient needs medical treatment in a specialty area not covered by a Magna physician, they are referred outside of the Magna network for care, and Magna pays all referral costs on a fee-for-service basis. Magna does not own any hospitals itself, instead contracting services from several local hospitals. Magna's CEO has retained McKinsey to help determine what is causing the declining profitability and how Magna might fix it. Question #1: What key areas would you want to explore in order to understand Magna's decline in profitability? Possible Answer: Some possible areas are given below. Great job if you identified several of these and perhaps some others. 1. Magna's revenues: Price paid by employer for employee health coverage Number of employees covered by Magna 2. Magna's costs (or fixed and variable costs) Magna's main cost components consist of administrative (non-medical) and medical costs (e.g. hospital, drugs, outpatient care) Outpatient costs an be split into internal physician costs versus external referral costs 3. Magna's patient base demographics/overall risk profile which may affect medical costs Question #2: The team discovers that the demographics of Magna's subscribers have changed significantly in the past 5 years, from majority industrial workers/laborers to majority office employees. Knowing this, are there any specific areas you would investigate first? Possible Answer: We are looking for a few responses, similar to the ones below: Claim costs, as the change in the subscriber base will change the profile of diseases (e.g., more heart disease/stress and less work related injury). External referral costs, due to the change in the disease profile for which they have in-house competency Question #3: After reviewing the basics of Magna's business, your team believes that one of the root causes of Magna's financial problems is how it manages medical costs, particularly the cost of referrals to specialists outside of its physician network. Your team has gathered the following information on Magna and its primary competitor, Sunshine HMO: Magna health case questionWhat are the most likely reasons that the average cost of referral at Magna is higher than at Sunshine? (At this point you should feel free to offer hypotheses, and you could ask your interviewer questions to clarify the information) Possible Answer: Although there are a number of possible responses, you might have the following suggestions: Referral pricing: Magna might be paying more than Sunshine for specialist services (e.g. its outside contracts with oncologists might be at higher rates than Sunshine's contracts). Number of referrals: Magna's physicians might have different practice patterns than Sunshine physicians, i.e. they may be less comfortable treating heart disease patients or have different training/protocols. Mix of specialties: Magna's mix of specialties that requires referrals (cardiology and neurosurgery) are probably more expensive specialties (than cardiology and psychiatry, Sunshine's referral specialties). Mix of patients: Magna has sicker or older (>65) patients (individuals over 65 are more likely to need medical care in the specialty areas outside of Magna's network, particularly cardiology). Question #4: What analyses would you do if the things you suggest were contributing to this problem? Possible Answer: You might take the following approach, where we've outlined different areas of analysis: 1. Referral pricing: Gain data on prices currently being paid by Magna for a sample of common specialties Gain similar data for a competitor if possible for an industry average (perhaps through interviews with non-Magna specialists) 2. Number of referrals: Interview Magna physicians and non-Magna physicians to see if any obvious behavioral differences exist Consult industry publications on this issue 3. Mix of specialties: Check number of referrals by specialty for Magna and estimate similar for Sunshine. Interviews with external specialties used by Sunshine may help again here 4. Mix of patients: Compare demographic data for Magna and Sunshine: should be easy to obtain from Magna; a scan of the employee schemes covered by Sunshine should give a good general picture of their demographic profile. See if Magna's referral cost has increased in line with the change in demographics of the subscribers Helpful Tip: In giving the answer, it's useful if you are clear about how the analysis you are proposing would help to answer the question posed. Question #5: Magna's CEO has a hypothesis that Magna is paying too much in cardiology referral costs for its patient population. He asks the McKinsey team to look at Magna's cardiac patient population more closely and tell him how many referrals he should expect on an annual basis. Assume the following: Magna has 300,000 patients in any one year 20% of its patients are age 65 or older In the U.S. , patients with serious heart disease visit specialists (cardiologists) on average of 5 times per year You should always feel free to ask your interviewer additional questions to help you with your response. In this case, you should recognize the need to know the prevalence rate of serious heart disease to complete this calculation. Once asked, your interviewer would provide you with the following information: The prevalence rate of serious heart disease in the 65+ population is 30% The prevalence rate of serious heart disease in the under age 65 population is 10% Possible Answer: Based on the correct calculations, your response should be as follows: Magna should expect 210,000 cardiac referrals annually based on its patient population. You should have approached the calculations as follows to arrive at that answer: 300,000 total patients 20% x 300,000 = 60,000 patients age 65+ 18,000 x 5 = 90,000 referrals per year 240,000 Magna patients under the age of 65 240,000 patients x 10% = 24,000 patients under age 65 with serious heart disease and 24,000 x 5 visits per year = 120,000 visits per year total 90,000 + 120,000 visits per year = 210,000 total Magna patient external cardiology visits Helpful Tip: While you may find that doing straightforward math problems in the context of an interview is a bit tougher, you can see that it is just a matter of breaking the problem down. We are looking for both your ability to set the analysis up properly and then to do the math in real time. Question #6: When the team tells Magna's CEO that based on Magna's patient population he should expect about 210,000 cardiology referrals a year he exclaims, "We currently pay for 300,000 annual cardiology referrals for our patient population!" Why might Magna's annual cardiology referrals be significantly higher than U.S. averages? What would you do to try to verify if any of these were a key cause of this problem? Possible Answer: There are a number of answers to these questions, and you are on the right track if your responses included some of the ones below: The prevalence rate of heart disease in Magna's patient population is higher than average. To see if this was a cause of the problem, McKinsey should audit the internal data on heart disease prevalence and compare it to US National data. Magna's primary care physicians are referring patients who do not have serious heart disease to specialists. The team should interview specialists to get their opinion, or follow through a sample of patients who were referred. Primary care physicians are not comfortable (e.g., they are poorly trained or inexperienced) treating cardiac patients, even those with minor problems; they want to avoid malpractice suits. McKinsey should interview Magna physicians and institute an external review. Magna doesn't have clear guidelines on when physicians should be referring patients to specialists (or if guidelines exist, physicians are not complying with them). The team should gain an expert opinion on the current guidelines to see if this was a key cause of the problem. There are no incentives or penalties to prevent physicians from referring patients with less serious problems to specialists. In order to verify this is a key cause of the problem, the team should review incentive schemes if they exist. They should also compare similar companies/situations (e.g. prescription control mechanisms, etc.). Helpful Tip: We would not expect you to come up with all of these answers, but we hope some of your answers head in the same direction as ours. Yours may bring some additional insights. In either case, be sure that you can clearly explain how your reasons will bring you closer to the why the referrals might be higher. Question #7: At this point in the study, you bump into Magna's Head of Health Services in the corridor. He is responsible for all matters related to the provision of services to subscribers, both inside and outside the Magna Network. He asks you if you have made any progress. How would you respond? Possible Answer: The ability to come to a logical, defensible synthesis based on the information available at any point in an engagement is critical to the work we do. Even though we'd consider ourselves to be early in the overall project at this point in the case, we do want to be able to share our current perspective. One ideal answer would include the following points: Findings We have investigated all the drivers of profit for Magna. Although there is likely to be room for improvement in a lot of areas, it seems the claims cost is a big area for improvement. Relative to the market and to competitors, Magna seems to have high claims cost per patient. Our initial indication is that there may be highest room for improvements in the cost of referrals outside the network. There are a number of reasons as to why this may be happening (list as in previous question). Next Steps We are working to pin down the most significant reasons why Magna has high claims cost per patient. We are going to be looking into other areas such as reduction potential in other costs, as well as improvement potential in terms of premiums or other sources of revenue. Helpful Tip: Think about the person you are talking with, and how best to communicate the findings you have come up with so far. Question #8: After some additional investigation, your team decides that changing the behavior of Magna's primary care physicians has potential to reduce cardiac referral costs while maintaining high quality care. The team believes than introducing some sort of incentive plan for physicians might help reduce the referral rate. You propose the following pilot plan: Magna pays bonuses of $100,000 per year to each of the 10 primary care physicians with the lowest cardiac referral rates consistent with good patient outcomes. Magna increases overall fees paid to primary care physicians to handle more of their patients' basic cardiology needs. Overall fee increases would total $1 million. How many fewer cardiology referrals will Magna need to have in order to recoup the cost of the pilot incentive plan? For simplicity's sake assume: The cost of a cardiology referral is $200 Magna currently has 300,000 cardiology referrals per year Possible Answer: If the incentive plan reduces cardiology referrals by 3.3% or 10,000 referrals, Magna will recoup the cost of the incentive plan. One potential approach to the calculation: $1 million + (10 * $100,000) = $2 million for incentive plan $2 million/$200 =10,000 referrals 10,000 referrals/300,000 total referrals = 3.3% reduction would pay for incentive program Question #9: Your team projects that the incentive plan has the potential to reduce referrals by 5% in its first year, and an additional 2% in its second year. If these projections are correct, by how much would Magna's referral costs be reduced over a two-year period with this program? Possible Answer: Referral costs would be $4.14 million lower in the second year. Over the two years Magna would save $7.14 million. One potential approach to the calculation: Year 1 Savings with Program 300,000 total referrals 5% reduction in referrals =15,000 referrals 15,000 x $200 = $3.0 million in savings in year 1 Year 2 Savings with Program 285,000 total referrals 2% reduction in referrals = 5,700 referrals 5,700 x $200 = $1.14 million in savings $3 + $1.14 = $4.14 million in savings Therefore, total cumulative savings over the 2 years = Year 1 savings + Year 2 savings = $3.0m + $4.14m = $7.14m. Question #10: Your team presents its physician incentive proposal to Magna's CEO. The CEO, in consultation with his Medical Director, agrees that this is feasible and says that they will pilot it for cardiac referrals. At the end of the meeting the CEO says, "I like the work you've done, but it's not enough to address our current financial situation. Physicians are professionals who care deeply about patient care and I think there's a limit to how much cost we can expect to reduce utilizing financial incentives exclusively. Besides cardiac financial incentive programs, what other ideas should we consider to reduce the cost of Magna's specialist referrals?" Based on what we have discussed today, and any other ideas you might have, how would you respond to the CEO? Possible Answer: This question is a good one for demonstrating creativity because there's a long list of possible ideas. You might give the following response: Pursue additional ways to change physician behavior: Provide training on how to treat patients with minor or stable medical problems Define and clarify medical guidelines for referrals (e.g., establish a medical committee to define the difference between "serious" and "minor" heart disease) Institute peer review committee charged with approving a subset of referrals (e.g., those that are considered "high cost,") Spend time investigating "outlier" physicians (i.e., those who seem to refer patients to specialists at much higher rates than others) to determine how widespread the referral problem is and whether simply focusing on a few physicians will dramatically reduce referral costs Determine whether Magna can reduce referral costs in the other medical areas where it does not have specialists (i.e. neurosurgery) Look at the contracts Magna has for specialist services to determine if it is paying too much relative to competitors Consider whether bringing cardiology, neurosurgery, and oncology specialists in-house (i.e., within Magna) might reduce cost Helpful Tip: You may have a slightly different list. Whatever your approach, we love to see candidates come at a problem in more than one way, but still address the issue as directly and practically as possible.

Case Interview Question #00167: Economics What factors determine the world price of crude oil?

Consider the influence of the capital markets and how future spot oil prices and speculation in the market impact international oil prices. Prices in general economic terms are a function of crude oil pricessupply and demand in the market. Demand: Explore the factors aside from price that would effect demand such as new technology, import quotas, wars, etc. It turns out that global demand for oil is inelastic meaning that changes in prices have less of an effect on quantity demanded. If this is the case, fluctuations in supply will have a greater impact on the price. Supply: Examine the impact of the various regional suppliers and their cost structures (for example, high cost in the U.S., low cost in the Middle East). At this point, you may recommend drawing a simple graph. The low cost producers such as Saudi Arabia would be at the lower left end of the supply curve while high cost producers such as the U.S. would be far to the right on the quantity produced x-axis. Thus, we can conclude that Saudi Arabia, assuming that it has the capacity to produce more than it currently is, controls the price of oil. However, its production is limited by OPEC rules. If, however, they use their excess capacity to control price (as in the case of the Persian Gulf War), the pricing power lies in their hands. Oil prices did not skyrocket during the war because Saudi Arabia promised to increase production to a level that eclipsed the global pre-war level. Another Possible Answer: The international price of oil is determined by three major factors: demand, supply, market sentiment. Demand Side Factors: Weather. Cold weather increases consumption. The world is getting hotter. The 14 hottest years in history have been in the last 25 years. The warmer the climate - the less oil is consumed for heating, but the more oil is consumed for air conditioning. Economic growth - The stronger the growth, the more oil is consumed (mostly for industrial purposes). The incredible economic development of countries like China and India and the emergence of car-owning middle classes in many developing countries enhanced demand and contributed to the current crisis. Ecological concerns and economic considerations lead to the development of alternative fuels and the enhanced consumption of LNG (Liquefied natural gas) and coal, at oil's expense. Even nuclear energy is reviving as does solar energy. Wars increase oil consumption by all parties involved. Supply Side Factors: Oil exploration budgets are growing and new contracts have just been signed in the Gulf area (including Iraq), Brazil, and Canada. The more exploration, the more reserves are discovered and exploited, thereby increasing the supply side of the oil equation. Lifting of sanctions on Iraq, Iran and Libya will increase the supply of oil. When there is an economic crisis in certain oil producers (Russia, Nigeria, Venezuela, Iraq) it forces them to sell oil cheaply, sometimes in defiance of the OPEC quotas. This was the case in the late 1990s. OPEC agreements to restrict or increase output and support price levels should be closely scrutinized. OPEC is not reliable and its members are notorious for reneging on their obligations. Moreover, OPEC members represent less than half the oil produced globally. Their influence is limited. New oil exploration technology and productivity gains allow producers to turn a profit even on cheaper oil. So, they are not likely to refrain from extracting and selling oil even if its price declines to 5 US dollars a barrel. Privatization and deregulation of oil industries (mainly in Latin America and, much more hesitantly, in the Gulf) increases supply. Recent moves in countries like Venezuela, Russia, and Bolivia to re-nationalize their oil industries and unrest in countries like Nigeria raise global oil prices owing to uncertainty and increased political risk. Market Sentiment: The mere belief that oil demand will increase dramatically at some point in the future can result in a dramatic increase in oil prices in the present as speculators and hedgers alike snap up oil futures contracts. Of course, the opposite is also true. The mere belief that oil demand will decrease at some point in the future can result in a dramatic decrease in prices in the present as oil futures contracts are sold (possibly sold short as well). Price volatility induced by hedge funds, future contracts and other derivatives trading has increased lately. But, as opposed to common opinion, financial players have no preference which way the price goes, so they are neutral.

Case Interview Questions #00013: You are visiting your client Bridgestone Golf who sells golf balls in the United States. The company is a subsidiary of Bridgestone Sports Ltd, and is based in Covington, Georgia, United States. Its parent company is multinational rubber conglomerate Bridgestone Corporation. The client is primarily bridgestone golf ballsknown for their golf balls, and claim to be the number one golf ball producer in the world. Having had no time to do background research, you sit on the plane wondering what is the annual market size for golf balls in the U.S. and what factors drive demand for golf balls. Your plane will land in less than fifteen minutes. How do you go about figuring out these questions?

Golf ball sales are mostly driven by end-users. 1. In order to estimate the number of end users: Take the US population of 300 million; assume that people between 20 and 70 play golf (about 2/3 of the population, or 200 million) and estimate what proportion of these people ever learn to play golf (guess 1/4) which reduces the pool to 50 million. 2. Now, estimate the frequency of purchase: If the average golfer plays twenty times per year, and requires two balls per time, that's 40 balls per person. Multiply that number by the 50 million people, resulting in a 2 billion gold ball market.

Case Interview Questions #00131: PRICING STRATEGY. THINK ABOUT THEORY Your client is General Electric Company (NYSE: GE), a multinational conglomerate corporation headquartered in Fairfield, Connecticut, US. The company operates through four segments: Energy, Technology Infrastructure, Capital Finance and Consumer & Industrial. In 2011, Fortune magazine ranked GE eternal light bulbGE the 6th largest firm in the U.S. Recently, researchers in GE's Energy Division have just developed a new product: a new light bulb that can last eternally. Your job is to help them go to market by defining their pricing strategy. The new light bulb cost $1 billion in total to develop.

Me: Are the new light bulbs different from conventional light bulbs in any other way? Are the light bulbs intended to replace regular incandescent light bulbs or fluorescent light bulbs? Interviewer: You can assume that the new light bulbs can be used to replace both incandescent and fluorescent light bulbs. You can further assume that the new bulbs are perfect replacements for these types of bulbs (i.e., they are available in two different forms, one that is exactly like any other incandescent bulb, and one that is like any other fluorescent bulb). Me: In order to determine the optimal pricing strategy, we'll need to look at both microeconomic and marketing theory. First, it may be useful to determine the upper and lower limits on the price GE can charge for its new light bulbs. In general, price is bounded by two things: the product's economic value to the customer (EVC) and the company's average cost in producing the product. The EVC sets the upper bound in price since a person will not pay more than the product is worth to her, and the average production cost sets the lower bound since the company can not earn economic profits if the price is below this point (in the short run, however, the company will want to produce as long as price is above average variable costs since this yields a positive contribution to fixed costs). The optimal price must fall somewhere within this range. Interviewer: How would you determine the lower and upper limits in price? Me: The average total cost of production can be obtained by considering fixed costs for the product (e.g., overhead and administrative costs), plus manufacturing costs, plus distribution costs, plus selling costs, and so on. Of course, the average cost will vary with the level of production. Generally, the average cost function is U-shaped (where the x-axis measures quantity and the y-axis measures average cost). Note that the average total cost of production is independent of the $1 billion development costs. This makes sense since this is a sunk cost. The sunk cost does affect the overall return on investment (ROI) and the internal rate of return (IRR) for the project, however. The EVC can be computed as follows: First, we assume for simplicity that the resale value of the new light bulb is negligible after it has been used for many years (this is akin to any other old household item). We further assume that the average person will be able to use one of the new light bulbs for 50 years before it is discarded (either because it is accidentally broken or because the person dies and his belongings are disposed of). Finally, we assume that a normal light bulb lasts an average of 6 months and costs $.50. Now, to compute the EVC we need to determine how much one of the new light bulbs will save a person. Since we assume that the new light bulb has an effective life of 50 years, it will save a person $1 a year from having to buy two old light bulbs for 50 years. Thus, the EVC is approximately the net present value of a $1 annuity for 50 years (to be more accurate, we would have to consider the economic value of the time savings from having to buy and replace normal light bulbs, the reduced risk of being electrocuted from not having to frequently changing normal bulbs anymore, and so on. We assume that this is negligible. At the same time, however, we must also realize that the new, significantly more expensive light bulb may be accidentally broken prematurely (e.g., while moving to a new house), resulting in an economic loss for the customer. The probability of this should be considered in the EVC.). Interviewer: Good. Now how would you determine the optimal price? Me: From microeconomics theory we know that the optimal, profit-maximizing price is given by the equation: P = ( Ep / (1 + Ep) ) x MC, where Ep = price elasticity of demand for the new light bulb, MC = marginal cost of producing the new light bulb Interviewer: How would you get the elasticity and MC data that you need to use the optimal price formula? Me: The marginal cost of manufacturing, packaging, distributing and selling the new light bulb can be obtained by performing a cost study of these processes. For instance, the marginal costs associated with manufacturing will include the costs of raw materials, direct labor, and energy. Of course, the marginal cost will vary with the level of production. In general, the marginal cost curve is roughly U-shaped. The elasticity function is more difficult to obtain. Generally, this is hard to derive in real life, especially for a new product that lacks past sales data. However, GE may be able to estimate the demand and elasticity function for the new light bulb based on its historical sales data of normal light bulbs. Using this data in a regression analysis, it can determine what the key drivers of demand are. For instance, it can perform a regression analysis with sales quantity as the dependent variable and price and bulb lifespan as the independent variables (the exact type of regression model will need to be determined - i.e., logarithmic, linear, exponential, etc.). The elasticity function may also be estimated by conducting a survey of potential customers of the new light bulb. In this survey, customers can be asked what quantities they would purchase the new light bulb at different price points. This data can then be used to derive the elasticity function. Interviewer: This sounds like a lot of work. Do you really need to do all of this to determine the optimal price? Me: No, you're right. The optimal price can be accurately estimated. We know that at the industry level, demand for light bulbs is highly inelastic since light bulbs have become a necessity and there are few substitutes for them (cross-elasticities are low). At the same time, however, light bulbs are a commodity. Thus, at the firm level, there is nearly perfect competition for light bulbs, and demand is perfectly elastic for any single firm. As a result, the optimal pricing strategy for GE is to price its new bulbs slightly below the EVC for the new bulb (which is equivalent to pricing is slightly below the market price for conventional bulbs) since this will provide consumers a savings in using the new bulb (this assumes that the average production cost for the new bulb is below this price level. If it is not, it is not economical for GE to produce and sell the new bulb). If GE were to price its bulbs above its EVC, consumers would have no incentive to purchase it. If it were to price the new bulb at the EVC, the new bulb would offer no advantages to a conventional bulb, and it would just be another commodity bulb. As a result, it would not allow GE to significantly increase sales and profits. GE needs to consider a few other issues in its pricing strategy. First, it should price its new product low initially to induce trial. Second, severe cannibalization of its conventional bulbs is likely to result. Thus, GE needs to ensure that the sale of its new bulb will offer a higher contribution margin than that from the sale of its conventional bulb. Lastly, GE needs to consider the industry's competitive reaction. Since the industry is a commodity market, P = MC, and thus, it is unlikely that competitors can afford to compete by lowering the price of their products. They may, however, attempt to build their brands to make their product less of a commodity. Interviewer: How would your analysis be different for GE's business customer segment (i.e., for businesses that use the new bulb to replace fluorescent lights)? Me: In our consumer analysis, we assumed that the economic value due to the time savings from not having to buy and replace new bulbs is negligible (primarily because the opportunity cost is negligible - what is the opportunity cost of saving 2 minutes to pick up light bulbs while at the grocery store or from saving 2 minutes at home installing the bulb?) With business customers, however, this is not the case. The elimination of the need to replace bulbs periodically will save businesses money from having to hire maintenance personnel to do this. Thus, the EVC for businesses will be higher than that for consumers, and GE can charge businesses a higher price. In addition, we also assumed that the resale value of a new bulb that has been used for many years is negligible for the consumer since, aside from garage sales, it may be difficult for the individual seller to locate a buyer (this is currently changing as a result of the Internet, though. But, then again, how many people would be willing to pay a non-negligible amount of money for an old household item, such a hammer or an old mirror, both of which can theoretically last a long time like the new GE light bulb?). This is not the case with business customers, however, since the resale market for old business furniture is relatively strong. In addition, it is likely that the expected lifetime of a bulb used in a business environment will be longer than that used in a consumer's home. The reason for this is that bulbs are generally fixtures in the office building even as the occupants in the building change. Thus, the expected lifetime of the new bulb in a corporate office is likely to be about the same as that of the building. These factors will allow GE to charge an even higher price to its business customers. Interviewer: Good. I think that's all I wanted to cover with this case. Do you have any question for me about the firm?

Case Interview Question #00327: ENTER NEW MARKET, CHANGING DEMAND DUE TO SEASONS Your client Dr Pepper/Seven Up Bottling Group is a large international soft drink bottler. Currently it is the third largest bottling company in the United States after Pepsi Beverages Company and Coca Cola Enterprises. The company is a wholly owned subsidiary of Plano, Texas based Dr Pepper 7upsoft drink company Dr Pepper Snapple Group (NYSE: DPS). Recently, the client is considering pursuing growth overseas in emerging markets. In particular, the management wants to know whether it is a good move to enter the Indian market. You have been hired to advise them on their market entry plan. What are the concerns and potential issues in doing this? Should the client enter the Indian market or not? And why?

Possible Answer: 1. Industry: What are the profit margins in the soft drink industry? Upon inquiry I found that profit margins are particularly high in the soft drink industry (~30%). I then mentioned potential Cultural Issues associated with entering Indian market (flag burning, and hatred of Western commercialism in India). This point was duly noted, but the interviewer directed me towards focusing on the economics. In the end you should only do it if it is profitable to do so. 2. Costs: What are the upfront costs? What is the selling and distribution system? What is the account structure? What are the distribution costs? Can we get access to all of the distribution channels, or do we need to partner with someone who has a distribution network set up already? We discussed up-front cost to enter the market in terms of infrastructure of plants, bottling lines, warehouses, delivery trucks and coolers. An analysis of the cost structure resulted in a reasonable operating cost structure but the level of investment required to become a significant player might be prohibitive, despite the attractive 30% operating margins of soft drinks. 3. Customers: What is customer demand like? An analysis of the customer led to the discovery that there probably was a latent consumer demand for cold drinks especially in the summer, highlighting the importance of visi-coolers (i.e., refrigerators with glass doors so the consumer can see the product). In addition the seasonality seemed too severe and would force our bottling costs way up in the winter, with idle capacity sitting around. Recommendations for Client: Do not enter the Indian market due to high initial costs and high seasonality of consumer demand.

Case Interview Question #00732: Your client Toys "R" Us, Inc. is a global specialty retailer of children's toys and baby products headquartered in Wayne, New Jersey, United States. The company operates more than 872 Toys "R" Us and Babies "R" Us stores in the United States, more than 715 international stores and Toys R Usover 180 licensed stores in 35 countries and jurisdictions. It also operates a portfolio of e-commerce sites including Toysrus.com, Babiesrus.com, eToys.com, and FAO.com. On July 21, 2005, a consortium of Bain Capital Partners LLC, Kohlberg Kravis Roberts (KKR) and Vornado Realty Trust invested $1.3 billion to complete a $6.6 billion leveraged buyout of the Toys "R" Us company. After the recent buyout by private equity, the client Toys "R" Us has been pursuing aggressive growth targets across all three of its divisions. The e-commerce division has about $400 million in total revenue and a major area of growth has been identified within Shipping & Handling (S&H) of customer orders. Your client offers its customer's two Shipping & Handling options: "2nd Day Expedited" or "Ground", and each of those are further divided into "Lower 48 States" or "Alaska & Hawaii" depending on the order destination zip code. Annual S&H revenue from each product offering is as follows: Ground to Lower 48: $32 million Ground to AK & HI: $180,000 Expedited to Lower 48: $3.73 million Expedited to AK & HI: $90,000 The e-commerce division of Toys "R" Us hopes to increase Shipping & Handling (S&H) profits by ~115%. You are part of a consulting team studying their S&H economics and are asked to make a recommendation to impact immediate growth and meet their goals. What would you recommend?

Possible Answer: I. Areas of Discussion A. First, explore the revenue and pricing structure: 1. Select SKU's (items) in inventory have fixed S&H prices for each of the four delivery options regardless of delivery destination. 2. 21% of all orders include SKU's using these fixed prices. 3. The majority of inventory is priced according to the weight of the order against a "weight table". Weight table pricing is as such: Ground shipments to the Lower 48 states begin at $4.84 for the first 1 lb and increases at $0.85 for each additional pound. Ground shipments to AK & HI begin at $6.84 for the first 1 lb and increases at $0.85 for each additional pound. 2nd Day shipments to the Lower 48 states begin at $10.98 for the first 1 lb and increases at $0.99 for each additional pound. 2nd Day shipments to AK & HI begin at $20.98 for the first 1 lb and increases at $0.99 for each additional pound. This weight table is a blended price that aggregates the zone-based cost table that the carrier charges and adds a 25% markup. 4. 73% of all orders are priced against the weight table. Another 6% of orders include a mix of items with fixed price S&H and weight table pricing. B. Then, guide the interviewee to explore items with Fixed Pricing: Further analysis into the SKU's that have fixed shipping prices shows that 44 items represent 10% of merchandize revenue of all fixed price items and represent 59% of the losses in that segment. C. Explore the cost structure: Your client uses a single package delivery vendor to ship all orders. The vendor charges your client for each order based on the following factors: Distance. The lower 48 states are categorized into 8 zone distances, with 1 being the closest and 8 the furthest. Alaska & Hawaii are divided into 2 zones; Metro and Remote. Size. Small packages (less than 3 cubic feet) are billed based on its weight. Large items are billed based on volumetric dimensions. D. Have the interviewee brainstorm about other shipping cost drivers per order: Number of Shipments. A large order with smaller items is likely to be more "combinable" and may ship in fewer boxes than one with larger items. "Over-boxed" items. Manufacture's packaging of some items is not durable for cargo shipping and therefore needs to be put in another box, or "over-boxed". E. Explore the following questions after graphs have been shared with the interviewee: Question #1. The largest loss bucket in S&H is Ground shipments to the Lower 48 states. Why should your client not increase its S&H prices for this group? Why should they increase prices on the other three S&H offerings? Possible Answer: Ground shipping also constitutes the largest portion of your clients business and increasing prices could negatively affect demand for items and orders. Expedited shipping is a premium product and the customer is likely to be less elastic to an increase in price. Therefore an increase in price might not impact demand too much. Alaska and Hawaii constitutes a small fraction (less than 1%) of your client's total business while contributing to ~5% of losses. Unless it is a business decision to maintain a presence in that region at such a cost, there is no reason to do so. Question #2. Your client's shipping prices are on average 25% more than their vendor's price table. Why do you think your client is seeing egregious losses in certain categories? How can they tackle this? Possible Answer: The egregious loses are primarily due to "over-boxing" of items due to the limited number shipping boxes. The client would need to better understand the demand on various items, their dimensions, what items are most frequently combined and then determine the optimal assortment (number and dimensions) of shipment cartons they use. This would be a longer-term solution. Question #3. If you were to recommend a new pricing strategy that raises prices, how much would your business be at risk? How much of you estimated losses can you save? Possible Answer: The percentage of business at risk is the ratio of the total merchandise dollars shipped through the segments that the candidate is recommending a price change to over the total business. If the interviewee recommends raising prices to Alaska & Hawaii, Expedited shipments and the 44 fixed price items the interview should identify approximately 11% of the business at risk (calculations below). Percentage of loss saved for the segments to AK & HI, Expedited shipments and the 44 fixed price items is approximately 30% (calculations below). Question #4. Should raising prices be a long-term solution? Why or why not? Possible Answer: Raising prices is not a long-term solution because it leaves your client at a competitive disadvantage. The long-term goals should be to reduce costs by further analyzing each of the cost drivers. Some longer-term solutions are: Conducting a "carton-size study" to determine the optimal assortment of boxes used. Look to expand the distribution network to also ship items from distribution centers on the east and west coast. Negotiate lower shipping costs with the vendor. II. Analysis (Calculations to be completed by interviewee.) A. Margin on each product is calculated with the following formula: (Margin On Wins + Margin On Losses) / Total Revenue In Category Margin on Ground to Lower 48: 13% Margin on Ground to Alaska & Hawaii: -142% Margin on Expedited to Lower 48: -17% Margin on Expedited to Alaska & Hawaii: -39% B. Percentage of business at risk: (Merchandise To AK & HI + Merch To Expedited + 10% × Total Merch With Fixed Price) / Total Mechandize Dollars = (200 K + 30,000 K + 1,800 K + 10% × 76,440 K) / 364 MM = 10.89% C. Percentage of Loss Saved: (Loss Margin To AK & HI + Loss Margin To Expedited + 59% × Loss Margin With Fixed Price) / Total Mechandize Dollars = (257.2 K + 1,010 K + 38.18 K + 59% × 892.6 K ) / 6,172.2 K = 29.68% 1. Business Shipped to Various Segments. Approximately 1/3 (~28%) of the business is shipped to customers at a loss Approximately 2/3 (~78%) of all business to Alaska and Hawaii (Ground & 2nd Day) are shipped at a loss Approximately 1/3 (~34%) of all orders that have items with a fixed price are shipped to customers at a loss 2. Gross Margin on Category Wins and Losses. Orders shipped by Ground to the Lower 48 states (which is ~91% of the business) accounts for ~79% of Losses Orders with only items with fixed S&H pricing (which is 21% of the business) accounts for ~14% of Losses Orders shipped Expedited to the Lower 48 states (which is ~8% of the business) accounts for ~16% of Losses Orders shipped by Ground to Alaska and Hawaii (which is ~0.5% of the business) accounts for ~4% of Losses Orders shipped Expedited to Alaska and Hawaii (which is ~0.05% of the business) accounts for ~0.6% of Losses III. Recommended Conclusion In the short-term the client can save ~30% of losses by: Increasing fixed S&H prices on 44 SKU's. Raise weight table prices for Ground orders to AK & HI and Expedited orders to both the Lower 48 states and AK & HI. The higher prices might leave the client at a competitive disadvantage and therefore the long term goal should be to reduce costs and return prices to normal. The client can look to reduce costs by: Conducting a "carton-size study" to determine the optimal assortment of boxes used Look to expand the distribution network to also ship items from locations in the east and west coast Negotiate lower shipping costs with the vendor

Case Interview Question #00252: PRODUCTION, KNOWING THAT THE ISSUE MIGHT NOT BE A PROFITABILITY QUESTION, There is this middle-sized rail wheel company in New South Wales in Australia. Its major business is to make steel wheels for railway carriages. Our client Pacific Equity Partners (PEP) is one of Australia's four largest buyout firms. PEP just bought this rail wheel company and they feel that rail wheelits performance can be improved. We have been called in to help. How would you go about analyzing the case?

Possible Answers: Candidate: Is it the production performance they are concerned about, or is the performance related to the revenues and costs? Interviewer: It's a production issue. Candidate: What's the issue? Interviewer: Not many wheels are being manufactured. Candidate: Not many wheels compared to the competitors, I assume. Interviewer: Yes. Candidate: Okay, how many wheels are we currently making? And how many are our competitors making? Interviewer: We don't have much information about our competitors except that they are making more wheels than we are. We currently make around 63,000 wheels per year. Candidate: Okay, so 63,000 wheels per year is approximately 5,250 wheels per month or 175 wheels per day. (I divided the monthly number by 30 to ease up my calculations). We need to make more than 175 wheels per day, right? Interviewer: Yea, how do we do that? Candidate: How are the wheels made currently? Like what is the wheel making process? Interviewer: We buy 1m x 10m bars of steel which are cut into 30cm slices called cheese. This cheese is melted and put into a milling furnace. While the hot cheese is spinning, heavy rollers squash it out into a wheel shape. This is then taken to a finishing area where it takes 2 days to cool down. Candidate: So each wheel takes 2 days to cool down? Interviewer: That is correct. Candidate: Is there something that can be done differently in this process? Like quicker cooling? Interviewer: No. Candidate: Could there be some process inefficiencies? Such as the steel bars not being the optimal shape for cutting, or steel impurities so that the cheese doesn't melt as fast? Do we buy enough steel to produce more than 175 wheels/day? Could there any wastage of raw material? Interviewer: Those are interesting observations. To your point, there aren't any process inefficiencies that could be the cause of our low output. But we do buy enough steel to produce more than 175 wheels/day. Candidate: That is interesting. So we do have enough raw material, but somehow aren't producing enough wheels. Could it be that some of the wheels being produced aren't usable? Interviewer: Good point! 5% of all wheels produced get rejected due to operational error. The wheel punching machine isn't calibrated well. Another 5% get rejected during testing due to structural problems in the wheels. Candidate: So only 90% of the wheels we produce are usable? Interviewer: That is correct. Candidate: Is the calibration issue due to human error? How can we improve the rejection rate? Interviewer: Yes. The calibration issue is due to human error. Some of the factory workers are not well trained to operate the machines. The structural problems we cannot address. So, why don't you tell me how we can improve the rejection rate? Candidate: Well, we can reduce the rejection rate from 10% to 5% if we can address the operational errors. As these errors are caused by poorly trained workers, we can implement training programs and quality control measures to ensure that lesser wheels get rejected. Interviewer: Okay, very good. How will this improve my monthly output? Candidate: So let's say we currently make x wheels/day out of which 10% are rejected. So 90% of x = 175. So x = 175/90% = 194. As the rejection rate has gone down to 5% due to our improvements, our output increases to 95%. So the total number of wheels we will make after these improvements is: 95% of 194 = 184 (as we cannot have fractional wheels). The number of wheels we can make monthly would be 184 * 30 = 5520. Interviewer: Very good. Now let's stick to our 175 number. We did a little more digging, and we found out that even with this 5% improvement, we were still behind our competitors in the number of wheels produced. Candidate: Why? Is the factory running under capacity? How many wheels can the factory produce in a day? Interviewer: We don't know how many wheels the factory can produce in a day, but it is definitely running under capacity. Candidate: Okay, so we know that we buy enough steel to produce more wheels. Could it be that the factory is not running 24/7? Interviewer: Excellent point. We asked around, and realized that each day, the factory experienced 8 hours of downtime: * 30 minutes due to MRO (maintenance, repair, and operations) * 190 minutes of unplanned maintenance due to machine breakdowns * 260 minutes due to production run changeovers where the operators change the wheel measurements Candidate: Which of these can we affect? Can we improve unplanned maintenance by buying new machines? Interviewer: We don't have the money to buy new machines. Candidate: What are production run changeovers? Why are wheel measurements changed? Can we shorten this? Interviewer: Well - we make wheels in 2 sizes. During a production run changeover, we recalibrate the punching machine to change the size of the wheel being produced. Candidate: Are there inefficiencies in this process? Let me elaborate with an example - Let's say we produce a big wheel and a small wheel. Suppose for the first quarter of the day, we make small wheels. Then we switch to making big wheels. In the third quarter of the day, we switch back to making small wheels - and so on. Each of these switches is contributing to wasted time during changeovers. If such inefficiencies exist, we can improve by only having 1 changeover. Lets make small wheels in the morning and big wheels in the evening. Hopefully, you get my drift =) Interviewer: Great point. We actually did end up facing a similar issue. Let's say, by improving the process , we can now reduce the time spent in production run changeovers to 80 minutes. How many more wheels will we make in a year? Candidate: Alright, let me have a couple of minutes to do some quick math. Before: 24 - 8 (downtime) = 16 hours makes 175 wheels. In one hour - we make 175/16 wheels. After: Downtime is now 30 + 190 + 80 = 300 minutes or 5 hours. (Means we have 3 more hours each day to produce wheels) So each year, we would make 3 * 365 * 175/16 = 11,976 (roughly 12,000) more wheels. Interviewer: I think that sounds about right. Now do you have any questions for me?

Case Interview Questions #00006: Your client Regions Financial Corporation (NYSE:RF) is one of the nation's super regional banks. A member of the S&P 100 Index, the company provides retail and commercial banking, trust, securities brokerage, mortgage and insurance products and services. Regions has more than $137 billion in assets as of 2010, Regions Bank branch making it the 22nd largest bank in the United States. Its banking subsidiary, Regions Bank, operates some 2,000 branches and 2,400 ATMs across a 16-state network in the South, Midwest, and Texas. The management team of Regions Financial has recently concluded that the old "local branch" way of business is no longer viable. Typically, this bank has canvassed its territory with small free-standing branches; however, the new age of electronic banking and commerce is changing all of that. They are considering replacing many branches with Calling Centers. Calling Centers offer both live and phone automated services that may be accessed by phone. The new Calling Centers would offer virtually all of the services currently offered through local branches plus some additional things. The question to you is: how would you go about setting up the engagement to determine the viability of this new concept? Specifically, what kinds of things would you investigate? And what hypothesis would you form?

Possible Answers: This is a very open broad-brushed case that involves "starting a new business service". There certainly is no right answer here; however this type of case occurs frequently. The following is a guideline of some things you should probably consider: 1. Market Analysis: What kinds of customers would be attracted to this no service? What kinds of customers would be turned off? (Hypothesis: younger people would be heavier users and more attracted than older) Of the people attracted to this new service, how profitable are they? How profitable are the people who are turned off by this service? (Hypothesis: older people have more money and thus are more profitable) 2. Added Revenues: What types of new services could be added to increase revenues? Automatic bill payment, fund transfer, etc. 3. Cost Savings: How much would it cost to establish a Calling Center and what are the risks involved? Do we have the expertise in-house to do this? How many branches could we close? Can we cut down on traffic to existing branches - thus requiring less tellers? Summary: This case probably is best to set up as a cost-benefit analysis. The number of new customers times the expected revenue from them plus the additional revenue generated by potential new services plus the cost savings must outweigh the forgone revenue generated by the customers you end up driving away.

INTERNATIONAL, RULE OF 72, SELL OFF, M&A Case Interview Question #00689: Our client Moldovan Coffins is a high-end coffin maker in the country of Moldova. Moldova, officially the Republic of Moldova, is a landlocked country in Eastern Europe located between Romania to the west and Ukraine to the north, east, and south. Moldova declared itself an independent handcrafted coffinstate with the same boundaries as the Moldavian Soviet Socialist Republic in 1991 as part of the dissolution of the Soviet Union. The total population of the country amounted to 4 million (2004 Moldovan census). The owner of Moldovan Coffins business has seen substantial change in his market in recent years and is contemplating the future of his business. Up until now, he has been in the business of building high-quality, handcrafted coffins largely by hand with a skilled labor force. Recently, however, he has become aware of a new technology that would allow him to build machine-made coffins with much less labor. Should he invest in this new technology? And should he even remain in the coffin-making business in the first place? Why or why not?

Possible Solution: Note that this case requires the interviewer to drive key points in the discussion. Allow the candidate to formulate a plan and then prompt him/her to consider each of the topics listed below. Question #1: What strategic alternatives should the owner of the coffin business consider? Possible Answer: If the candidate doesn't get all of this, help them along since we need to lay this foundation for the rest of the case - We need to decide firstly whether to stay in the coffin business at all and if so, whether he uses the new technology: Option 1: Sell the business to a third party Option 2: Sell the assets of the company and shut it down Option 3: Keep operating as is Option 4: Keep operating and invest in the new technology Question #2: How would you figure out the current value of the coffin business? Provide the following additional information if the candidate asks for it clearly and directly. Market Size - If the candidate asks for the size of the market, first make him/her brainstorm about different ways to determine market size. A good candidate should come up with at least 4 different ways, such as: Calculate from the market's total population, population growth, and birth rate. Review of death records for a period of time. Take sample of the number of obituaries in paper serving given population base. Calculate from total population, average life expectancy. Question #3: Now make the candidate calculate the market size, giving them the following data: Population of Moldova: 4 million Population Growth: 0% Average Life Expectancy: 75 years Age Distribution: assume a flat age distribution, i.e. same number of people at every age. Burial Customs: 75% of deaths are buried in coffins. Possible Answer: (4 million) x (1/75) * (75%) = 40,000 coffins purchased per year. Note that the candidate needs to quickly realize that every year, 1/75th of the total population will turn 76 and therefore (on average) will die. Question #4: Now make the candidate calculate the value of Moldovan Coffins' business, giving them the following data: Price - Coffins are priced at $5,000 for a hand-made high-end coffin. Costs - Material accounts for 10% of the direct cost, while labor accounts for the other 90%. COGS is $4,800 per coffin. Fixed costs for the business are $700,000 per year. Assume all assets are fully depreciated and ignore taxes. Competition - The client Moldovan Coffins has a 10% market share and a relative market share of about 1 (if asked, you may explain that relative market share is the ratio of the company's market share to that of its nearest competitor.) Market Trends, Regulation, etc. - If asked about any exogenous factors, simply tell the candidate to assume that the market is expected to continue as it currently is. Possible Answer: The candidate needs to calculate the value of the business now. This is a pure mathematical exercise. Margin per coffin = $5,000 - $4,800 = $200 Contribution Margin = $200 per coffin x 40,000 coffins x 10% market share = $800,000 Profit = Contribution Margin - Fixed Costs = $800,000 - $700,000 = $100,000 Assuming a discount rate of 10% (candidate can assume anything reasonable here as long as they are consistent later), a perpetuity with cash flows of $100,000 per year has a present value of $100,000 / 0.1 = $1 Million. So the current business is worth $1M whether they keep it or sell it. Question #5: So now what is the value of the company if it were shut down and the assets were sold? Additional Information to give if asked: Assets - Since the firm has been building coffins by hand, the fixed assets are essentially only the land and improvements. These are owned outright by the company. When the candidate asks for the value of the land, have them brainstorm ways that they might determine this. They should come up with at least 3 good ways, such as: Look for comparable real estate and determine recent selling price. Find comparable commercial real estate and determine the rent per square foot, then discount the cash flows generated by renting the property. Determine rate of appreciation for property in the area and then apply to book value of current land and improvements. Give the candidate the following information and have them calculate the value of the property: Book Value of Land: $20,000 Book Value of Improvements: $80,000 Years Owned: 48 Average Real Estate Appreciation: 6% / year Possible Answer: Using the "rule of 72″, a 6% growth rate will double the investment every 72/6 = 12 years. Since the property was held for 48 years, the current value will be $100K * (2 x 2 x 2 x 2) = $1.6M. Since the assets ($1.6M) are higher than the value of the discounted cash flows ($1M), then it would make more sense to liquidate the business and sell the assets. Question #6: What would the value of the company be if the owner invests in the new technology? Provide the following information if asked: Investment - Investing in the new technology will cost the firm $1M. Cost Savings - Material costs remain the same, but labor costs are reduced by 50%. Proprietary Nature of Technology - The new coffin-making technology is being offered for sale by a machine tool company, who holds the patent. They are not offering exclusivity to any customers (i.e. they will sell to Moldovan Coffin's competitors if possible). Competitive Threat - It is not known whether the competitors have acquired or are planning to acquire this new coffin-making technology. Customer Preferences - While the machine-made coffins are not "hand made", the quality perceived by the customer is the same or better. It is believed that the customer will be indifferent between the quality and appearance of a hand-made and a machine-made coffin. Brand Impact - The candidate may argue that a machine-made coffin might negatively impact Moldovan Coffin's brand. If so, ask them how they would test this (e.g. consumer research), but tell them to assume that it would have negligible impact. Possible Answer: Since Moldovan Coffins has no proprietary control over the technology, it is likely that competitors will also acquire it, resulting in an overall lowering of the industry cost structure. If this is the case, price will also fall as competition cuts price in an attempt to gain share. If we assume that gross margins remain the same, since the industry competitive structure has not changed we can calculate the new margin contribution as follows: Gross Margin = $200 / $5,000 = 4% Labor Cost = (4800 x 90%) x 50% = $2,160 Material Cost = 4800 x 10% = $480 COGS = $2,160 + $480 = $2,640 Price = $2,640 / (1 - 4%) = $2,718 Contribution Margin = $2,718 - $2,640 = $78 per coffin Profit/Loss = $78 * 4,000 - $700,000 = -$388,000 So the introduction of the new technology to the market might be expected to reduce industry profits, making this business completely unprofitable. Candidates could argue other scenarios, by assuming that the industry would be able to maintain higher margins than we have assumed here, so the answer may be different. They should recognize, however, that the introduction of this non-proprietary technology will significantly reduce industry pricing in the absence of some other form of price support (such as branding, collusion between players, etc.) 7. Conclusion A star candidate will see that his/her time is nearly up and will present a recommendation for the client without prompting. If the interview is within 3 minutes of the end, ask: "The owner just called and said he has an offer to buy his business. He needs to know whether he should take it right now." Possible Answer: Given the credible threat of the industry becoming unprofitable due to the introduction of this new technology, the owner should look to sell the company as soon as possible. Taking into account the assets of the firm and the present value of the expected cash flows of the business itself, he should attempt to liquidate the business and to sell the assets for around $1.6M. If the owner is unable to sell the business now, he can continue to operate the business as a cash cow, but should not invest in the business above what is necessary to keep it operating at its present level. He should expect the business to become less profitable as the industry moves to mechanization, and should eventually look to sell the assets of the company and close the firm. Comments: This case was given by McKinsey in one of their first-round interviews and is a typical "command and control" style McKinsey case. In this style of case, the interviewer allows the candidate to drive the case initially to explore possible routes to a solution. However, once the candidate has laid out a plan, the interviewer takes control and asks the candidate to solve a few specific problems before coming to the final conclusion. When giving this case, allow for some initial planning and brainstorming by the candidate, but then firmly take control of each of the "modules" described in the case. Try to move the candidate along through each of them, since in the actual interview only those candidates that complete all of the sections will be considered to have done well. This case tests mental horsepower and the ability to move to conclusions quickly.

Case Interview Questions #00065: OPEN ENDED INDUSTRY ANALYSIS You are the newest member on the management committee of McKinsey & Company, a well known top tier global management consulting firm. Eager to be accepted by your more senior peers, you volunteer to study the management and strategy consulting industry and propose Mckinsey BCG Bain Booz ATKearneya firm strategy for the 2010′s, which you will present to the committee at its next meeting. As you leave the meeting room, you begin to realize the enormous task to which you've committed yourself. Question #1. How do you evaluate the consulting environment and determine likely future scenarios? Question #2. What information do you use in this process? How is this information obtained? Question #3. What do you believe is most likely to happen in the consulting industry given your present knowledge? How did you arrive at this conclusion? Question #4. What strategy do you propose to the management committee?

Possible Solutions: This is one of the most difficult types of cases because the answers are completely unknown and will vary substantially depending upon the interviewee's knowledge of the industry. This is also an interesting case since the salience is likely to be high. As an interviewer you should feel free to add information on an as needed basis. When information isn't available, ask the Interviewee to develop his or her own hypotheses. What matters here is the thinking process, not necessarily the answer. Answer #1. A good place to begin is to evaluate the industry from a competitive analysis perspective, such as Porter's five forces. The following is an abbreviated analysis. Rivalry (low to moderate): management consulting is fragmented, with many players each holding relatively small concentration of total market. Firms act as competitive monopolists, and differentiate themselves by specialty, type of customer (Fortune 100 versus Fortune 1000 companies), reputation (McKinsey versus accounting firms), and the resources they employ (top MBAs versus all MBAs). Many companies are relationship driven with their customers, which limits competition and keeps prices high. Top tier firms in particular are able to have high price points. Potential Entry (moderate): there are no great barriers to entry into consulting; however, few new consulting firms truly compete in the top tier. It's possible new firms would enter if the industry were earning positive economic profits and if they faced certain imitability (e.g. the ability to recreate what the top tier firms do). Substitutes (moderate): companies can move the consulting process in house by hiring ex-consultants and bright MBAs. Buyer Bargaining Power (moderate to high): In the last decade the consulting market has boomed, with supply generally following demand, which lowers buyer power. However, it is appropriate to question the effect recession might have on consulting industry. It's possible that demand may decrease as companies quit expanding, which would reduce demand, give buyers more bargaining power, and push prices lower. Supplier Bargaining Power (low moderate): Major suppliers are the intellectual capital employed by firm (e.g. experienced consultants who bring in sales and new consultants who provide analytics). Must pay market price or risk losing suppliers. Other interesting points might explore the key success factors in the consulting industry. What sets top tier firms from middle ones? Do any firms have specific sustainable competitive advantages? How does the marketing mix differ among firms? Does your firm have any specific core competencies or advantages that set it apart from other companies? Determining likely future scenarios is more ambiguous. There are at least several key point: what effect will a recession have on consulting firms? Will top tier firms suffer differently from others? How will the mix of products demanded change (e.g. cost cutting studies rather than market expansion studies)? Will the consulting market continue to expand or suffer a cutback? Or, will certain geographical areas expand (Pacific Rim, Eastern Europe) faster than others? Again, the thought process is more important here than actual answers. Answer #2. Information gathering is a key reason companies use consultants. An interviewee should have a decent understanding of business information sources and how information is gathered. Information can be broken into two groups: secondary and primary. Usually one begins with secondary material, specifically, a complete review of published literature (a "literature search") pertaining to the study (e.g. journal and newspaper articles, investment bank research, specialized studies, books, etc.). This often points towards other good sources (e.g. industry experts, associations, major competitors, government sources, etc.). Hypotheses are often created from the secondary information. Primary research is then used to focus in on the key issues. This research includes telephone interviews, in person interviews, mailed questionnaires, focus groups, laboratory experiments, etc. Answer #3. This answer will depend upon the material covered in the first two. Ask the questions: What trends are likely? What is a positive scenario? A negative one? If you had any information at your disposal, how could you get a better handle on this issue? Answer #4. Again, there is no right answer here, so the interviewee may balk. However, you can provide some structure. What are the key success factors to succeeding in the industry? Is there any way to achieve sustainable advantage which cannot he duplicated by your competitors? Can you use non traditional methods to achieve competitive advantage, such as leveraging through technology. Given your firm's competitive strengths and core competencies, what is the best strategic route?

Case Interview Question #00528: PROFITABILITY, AIRLINE IN OTHER COUNTRY, JOINT VENTURE, HOW TO LOWER COSTS Air Arabia is a low-cost airline with its head office in the Sharjah Freight Center, Sharjah International Airport, in Sharjah, United Arab Emirates (UAE). The airline is the first and largest low cost airline in the Middle East. Air Arabia operates scheduled services to 46 destinations in the Middle air arabia airlineEast, North Africa, the Indian subcontinent, Central Asia and Europe from its base in Sharjah, 11 destinations in 10 countries from Casablanca, Morocco, and 5 destinations in 5 countries from Alexandria, Egypt. Air Arabia is a small airline company with a fleet of just 27 aircrafts. The company's performance is somewhat mediocre and the CEO of the company is looking for opportunities to improve the profits. The company doesn't have extra funds or avenues for obtaining funds to buy more airplanes or make major investments in the immediate future. How would you approach this situation?

Question #1: How can the client Air Arabia increase revenues? Ask for at least 5 different approaches. Possible Answers: Heavy marketing and advertising Better customer segmentation and promotions tuned to the needs of different segments Analyze the routes and schedules. Then look at possibilities to change routes and schedules to attract more passengers Develop a demand curve for each flight and work around pricing to attract more passengers (Yield management) Study all the points of customer interaction and explore opportunities to improve customer service (If candidate talks about this you may ask for examples) Join an existing alliance or partner with other airlines. Restructure the sales force to improve ticket sales effectiveness (Align their incentives, increase their selling time, establish key account managers and so on) Work on the frequent flyer program (if any) and improve the program to retain customers. Look at aspects like co-branding etc. Also may also want to look at miles and revenues as against only miles while rewarding passengers. Start in-flight sales of various items (food, gifts etc) Introduce vacation packages Enter into cargo transportation (if possible) Question #2: The CEO of Air Arabia is interested in entering into a partnership with a major airline that flies to the same destinations. The CEO wants recommendations on what they should consider when determining if they should enter into the alliance. What would you tell him? Possible Answers: What will be the benefits from this alliance? On what basis will the revenues be shared? How to share costs? - Costs like marketing, advertising and so on Will there be any issues to the brand image and positioning? For example, a low cost airline wouldn't want to have an alliance with a premium airline. Are there cultural differences between the employees of the airline companies? The staff of both airlines need to co-operate on a daily basis to make this work and cultural differences would lead to issues. How to integrate the two airlines' IT systems? Are they compatible? Question #3: The alliance is expected to improve the load factor of the planes. First, calculate the current profits per plane per flight. Give the following additional information to the candidate. Additional Information: Average flight distance of a plane = 1,000 miles Average number of seats per plane = 250 Fixed cost per plane per flight = $20,000 Revenues = $0.25 per passenger per flight mile Other costs = $0.10 per seat per mile (not per passenger) Current load factor = 80% (Provide information about current load factor only if the candidate asks for it) If the candidate asks for the definition of load factor, please say "Load factor is defined as the ratio of actual number of passengers in a plane to the total number of seats in a plane". Possible Answer: The candidate is expected to perform the following calculations first. Revenue per plane per flight = 80% * 250 seats * 1000 miles * 0.25 per passenger per flight mile = $50,000 Cost per plane per flight = Fixed cost per flight + Variable cost per flight = $20,000 + $0.10 per seat per mile * 250 seats * 1000 miles = $45,000 Profits per plane per flight = Revenue - Fixed cost per flight - Variable cost per flight = $50,000 - $45,000 = $5,000 Question #4: Now if the alliance would lead to a 50% increase in profits per flight, how much will the load factor increase? Assume the other data given previously remain the same. Possible Answer: Post-alliance profits per plane per flight = (1 + 50%) * $5,000 = $7,500 Also profits per plane per flight = revenue - cost = (Load factor * 250 seats * 1,000 miles * $0.25) - ($20,000 + $0.10 * 250 seats * 1,000 miles) = $7,500, solve the equation, new loand factor = 84% The other way to interpret this is, "If the load factor increases from 80% to 84% the profit per flight will increase by 50%" Question #5: The CEO also wants to know what areas to attack to reduce costs and needs some quick suggestions. What would you tell him? Possible Answers: Hedge fuel Eliminate routes or schedules with very low load factor Outsource aircraft maintenance Introduce technology and automation to reduce labor costs Outsource or offshore ticketing, customer support, billing, rewards and related functions Explore options to flying into airports where the gate costs may be cheaper

Case Interview Question #00659: Our client is Great Burger, a fast food chain that competes head-to-head with McDonald's, Wendy's, Burger King, KFC, etc. Currently, Great Burger is the fourth largest fast food chain worldwide, measured by the number of stores in operation. As most of its competitors do, Great Burger Dunkin' Donutsoffers food and "combos" for the three largest meal occasions: breakfast, lunch, and dinner. Even though Great Burger owns some of its stores, it operates under the franchising business model with 85% of its stores owned by franchisees. Individual franchisees own and manage stores under the "Great Burger" brand, pay franchise fee to Great Burger, but major business decisions, e.g., menu, look of store, are controlled by Great Burger. As part of its growth strategy, Great Burger has analyzed some potential acquisition targets including Canton, Massachusetts based Dunkin' Donuts (DD), a growing doughnut producer with both a U.S. and international store presence. Dunkin' Donuts operates under the franchising business model too, though a little bit differently than Great Burger. While Great Burger franchises restaurants, Dunkin' Donuts franchises areas or regions in which the franchisee is required to open a certain number of stores. Great Burger's CEO has hired McKinsey to advise him on whether they should acquire Dunkin' Donuts or not. How would you go about this case? Should Great Burger acquire Dunkin' Donuts as part of its growth strategy?

Question #1: What areas would you want to explore to determine whether Great Burger should acquire Dunkin' Donuts (DD)? Possible Answer: Some possible areas are given below. Great job if you identified several of these and perhaps others. a. Stand alone value of DD Growth in market for doughnuts DD's past and projected future sales growth (break down into growth in number of stores, and growth in same store sales) Competition - are there any other major national chains that are doing better than DD in terms of growth/profit. What does this imply for future growth? Profitability/profit margin Capital required to fund growth (capital investment to open new stores, working capital) b. Synergies/strategic fit Brand quality similar? Would they enhance or detract from each other if marketed side by side? How much overlap of customer base? (very little overlap might cause concern that brands are not compatible, too much might imply little room to expand sales by cross-marketing) Synergies (Hint: do not dive deep on this, as it will be covered later) c. Management team/cultural fit Capabilities/skills of top, middle management Cultural fit, if very different, what percent of key management would likely be able to adjust d. Ability to execute merger/combine companies Great Burger experience with mergers in past/experience in integrating companies Franchise structure differences. Detail "dive" into franchising structures. Would these different structures affect the deal? The McKinsey team started thinking about potential synergies that could be achieved by acquiring DD. Here are some key facts on Great Burger and DD. Exhibit 1 Stores Great Burger Dunkin' Donuts Total 5,000 1,020 North America 3,500 1,000 Europe 1,000 20 Asia 400 0 Other 100 0 Annual growth in stores 10% 15% Financials Great Burger Dunkin' Donuts Total store sales $5,500m $700m Parent company revenue $1,900m $200m Key expenses (% sales) * Cost of sales 51% 40% * Restaurant operating costs 24% 26% * Restaurant property & equipment costs 4.6% 8.5% * Corporate general & administrative costs 8% 15% Profit as % of sales 6.3% 4.9% Sales/stores $1.1m $0.7m Industry average $0.9m $0.8m Question #2: What potential synergies can you think of between Great Burger and Dunkin' Donuts (DD)? Possible Answer: We are looking for a few responses similar to the ones below: a. Lower costs Biggest opportunity likely in corporate selling, general, and administrative expenses (SG&A) by integrating corporate management May be some opportunity to lower food costs with larger purchasing volume on similar food items (e.g., beverages, deep frying oil), however overlaps may be low as ingredients are very different Great Burger appears to have an advantage in property and equipment costs which might be leveragable to DD (e.g., superior skills in lease negotiation) b. Increase revenues Sell doughnuts in Great Burger stores, or some selected Great Burger products in DD stores Great Burger has much greater international presence thus likely has knowledge/skills to enable DD to expand outside of North America Great Burger may have superior skills in identifying attractive locations for stores as its sales per store are higher than industry average, whereas DD's is lower than industry average; might be able to leverage this when opening new DD stores to increase DD average sales per store Expand DD faster than it could do on own-Great Burger, as a larger company with lower debt, may have better access to capital Question #3: The McKinsey team thinks that with synergies, it should be possible to double DD's U.S. market share in the next 5 years, and that Great Burger's access to capital will allow it to expand the number of DD stores by 2.5 times. What sales per store will DD require in 5 years in order for Great Burger to achieve these goals? Use any data from Exhibit 1 you need, additionally, your interviewer would provide the following assumptions for you: Doughnut consumption/capita in the U.S. is $10/year today, and is projected to grow to $20/year in 5 years. For ease of calculation, assume U.S. population is 300m. Possible Answer: You should always feel free to ask your interviewer additional questions to help you with your response. Possible responses might include the following: Market share today: $700M DD sales (from Exhibit 1) ÷ $3B U.S. market ($10 * 300M people) = 23% (round to 25% for simplicity sake) U.S. market in 5 years = $20 * 300M people = $6B DD sales if double market share: 50% * $6B = $3B Per store sales: $3B / (1000 stores * 2.5) = $1.2M Does this seem reasonable? — Yes, given it implies less than double same store sales growth and per capita consumption is predicted to double. Question #4: One of the synergies that the McKinsey team thinks might have a big potential is the idea of increasing the businesses' overall profitability by selling doughnuts in Great Burger stores. How would you assess the profitability impact of this synergy? Possible Answer: Be sure you can clearly explain how the assessment you are proposing would help to answer the question posed. Some possible answers include: Calculate incremental revenues by selling doughnuts in Great Burger stores (calculate how many doughnuts per store, times price per doughnut, times number of Great Burger stores) Calculate incremental costs by selling doughnuts in Great Burger stores (costs of production, incremental number of employees, employee training, software changes, incremental marketing and advertising, incremental cost of distribution if we cannot produce doughnuts in house, etc.) Calculate incremental investments. Do we need more space in each store if we think we are going to attract new customers? Do we need to invest in store layout to have in-house doughnut production? If your answer were to take into account cannibalization, what would be the rate of cannibalization with Great Burger offerings? Doughnut cannibalization will be higher with breakfast products than lunch and dinner products, etc. One way to calculate this cannibalization is to look at historic cannibalization rates with new product/offering launchings within Great Burger stores Might also cannibalize other DD stores if they are nearby Great Burger store-could estimate this impact by seeing historical change in DD's sales when competitor doughnut store opens nearby Question #5: What would be the incremental profit per store if we think we are going to sell 50,000 doughnuts per store at a price of $2 per doughnut at a 60 percent margin with a cannibalization rate of 10 percent of Great Burger's sales? Exhibit 2 Sales and profitability per store Units of Great Burger sold per store 300,000 Sales price per unit $3 per unit Margin 50% Units of DD sold in Great Burger stores 50,000 Sales price per unit $2 per unit Margin 60% Cannibalization rate of DD products to Great Burger products 10% Possible Answer: While you may find that doing straightforward math problems in the context of an interview is a bit tougher, you can see that it is just a matter of breaking the problem down. We are looking at both your ability to set the analysis up properly and then do the math in real time. Based on correct calculations, your response should be as follows: Incremental profit = contribution from DD sales less contribution lost due to cannibalized Great Burger sales = 50K units * $2/unit * 60% margin - 300K units * 10% cannibalization * $3/unit * 50% margin = $60K - 45K = 15K incremental profit/store Question #6: You run into the CEO of Great Burger in the hall. He asks you to summarize McKinsey's perspective so far on whether Great Burger should acquire DD. Pretend the interviewer is the CEO, what would you say? Possible Answer: You may have a slightly different list. Whatever your approach, we love to see candidates come at a problem in more than one way, but still address the issue as directly and practically as possible. Answers may vary, but here is an example of a good response: Early findings lead us to believe acquiring DD would create significant value for Great Burger, and that Great Burger should acquire DD We believe that our client can add $15 thousand in profit per Great Burger store by selling DD in Great Burger stores. This could mean $50 million in incremental profit for North American stores (where immediate synergies are most likely given DD has little brand presence in rest of world) We also believe there are other potential revenue and cost synergies that the team still needs to quantify Once our team has quantified the incremental revenues, cost savings, and investments, we will make a recommendation on the price you should be willing to pay. We will also give you recommendations on what it will take to integrate the two companies in order to capture the potential revenue and cost savings, and also to manage the different franchise structures and potentially different cultures of Great Burger and DD.

NON PROFIT, ENTER NEW BUSINESS Case Interview Question #00677: Our client Great Outdoors is a small not-for-profit organization near Philadelphia, Pennsylvania. It provides outdoor experiences including adventure therapy and wilderness therapy (the use of wilderness expeditions for the purpose of therapeutic intervention) to help pre-teen and young adolescents. wilderness therapy programThe nonprofit organization offers a range of different types of wilderness therapy programs, with a range of models and approaches. Originally, Great Outdoors is primarily concerned with providing 10-15 day primitive camping and hiking trips for troubled teenagers. However, recently a friend of one of Great Outdoors's founders asked the group to provide a shorter program for corporate executives, which was a great success in the first few iterations. The management of Great Outdoors has retained your consulting firm to make a recommendation on whether to expand Great Outdoors to create a full-fledged executive program. What would you recommend?

Read the Additional Information well before you give the case. Note that this case requires you (the interviewer or case giver) to drive key points in the discussion. Allow the candidate to formulate a plan and then prompt him/her to consider each of the topics listed below if the interviewee does not think of them themselves. Question #1: What issues should you consider? Possible Answer: Many different frameworks can work here as long as they drive to the main question and clearly account for the not-for-profit nature of the company. This case is really designed to test candidate flexibility since most people haven't practiced cases on nonprofits. A candidate who approaches the question as a standard business case without acknowledging that stakeholders are concerned with mission, emotional benefits, philanthropy, etc. will not succeed because they have not shown flexibility or good listening skills. Issues that should appear in the candidate's framework Economics of the two programs (don't let them drive this yet - we'll get to it later) Operational issues Major constraints on capacity (e.g. guide availability, permits, geographic reach, scheduling, etc.) Possible synergies from the two programs (e.g., offsetting seasonality of demand, economies of scale) Underutilized fixed costs (lots of idle time for guides or gear that could be utilized at no marginal cost) Mission: What is Great Outdoors' purpose? What is at risk from expanding? Are there constraints on engaging in commercial activity based on endowment or foundation restrictions? Will the current employees revolt if "profit" becomes a factor? Does organizational passion become compromised by "selling out"? Credibility issues by straddling profit/nonprofit worlds Competitive situation Question #2: Does expanding the executive program make sense economically? Provide the following Additional Information if the candidate asks for it clearly and directly. Additional Information: Competitors and Market Size - we do not have any specific information but believe we can sell out our capacity at current prices (no demand constraints). Competitors are very fragmented. Mission/Charter restrictions - we are not aware of any rules stipulating what the organization can or can't do - we don't anticipate losing funding based on a decision here Costs and revenues - Do not show this table directly to the interviewee; just share the numbers in a conversational way. They should create something similar in their notes. The candidate is expected to calculate the bold rows (Revenue, Subtotal, Subtotal, Total Margin) Revenue and Expenses Per Trip Executive Nonprofit # students per trip 15 8 Program fee per student $2,000 $800 Revenue 15 * $2,000 = $30,000 8 * $800 = $6,400 Days per trip 4 11 Costs (food, equipment, insurance, etc) per student per day $200 $125 Subtotal $200 * 4 * 15 = $12,000 $125 * 11 * 8 = $11,000 # of guides 4 2 Cost per guide per day $200 $150 Subtotal $200 * 4 * 4 = $3,200 $150 * 2 * 11 = $3,300 Total Margin $30,000 - $12,000 - $3,200 = $14,800 $6,400 - $11,000 - $3,300 = -$7,900 Question #3: At this point what is a preliminary recommendation? Possible Answer: Clearly the economics of the executive program are compelling. However it's critical to think about whether it is practical for the nonprofit organization to expand into this very profitable activity. Question #4: What would concern you about Great Outdoors starting a regular executive offering? Possible Answer: A good candidate should touch on these following factors: Competitor response - Competitors are very fragmented (the candidate should realize this means a response is not likely) Cost/Operational Savings - May be slight Loss of nonprofit status for fundraising, foundation charter restrictions, etc. - The company's lawyers believe this won't be an issue Organizational People who are motivated to help out a non-profit may be less willing to help if they feel that the organization has "sold out" to high-paying corporate customers Do corporate customers require different types of guides? Question #5: How would you deal with employees, donors, and other supporters who feel that working with corporate executives conflicts with Great Outdoors, an nonprofit organization's core values? Will working with profitable executive programs mean that traditional programs are compromised? Possible Answer: The candidate can brainstorm many different answers as long as they make sense. Some possibilities include: Sell the idea that every corporate program funds two traditional programs; by doing more corporate ventures we can reach more kids Executive programs are basically advertising Great Outdoors to a large pool of donor money; running exec programs is a good way to drive awareness among potential mentors, philanthropists, clients, etc. For some employees that are environmentally driven (think about the type of person that chooses to work as a guide) getting executives out to nature helps promote conservation and environmentalism. Question #6: What is your final recommendation? Possible Answer: This is a fairly open ended case that doesn't "crack" with any one insight; it is about assessing the way the interviewee approaches the process and how thoughtful they are about each issue. Good Answer: The organization should expand executive programs as there appears to be unsatisfied demand and it creates other benefits - demand balancing, scale benefits, etc. However communicating this decision to stakeholders correctly is critical to avoid alienating core supporters. Failing answer: Any answer that focuses only on the business and economic issues of the case without spending a considerable amount of time addressing the ideological and non-economic motives of nonprofit operators. Note: This case was given by McKinsey in first-round interviews and is a "command and control" type of case. In this style of case, the interviewer initially allows the candidate to drive the case and explore possible routes to a solution. However, once the candidate has laid out a plan, the interviewer takes control and asks the candidate to solve a few specific problems before coming to the final conclusion. When giving this case, the interviewer should allow for some initial planning and brainstorming by the candidate, but then firmly take control of each of the "modules" described below. Try to move the candidate along through each of them, since in the actual interview only those candidates that complete all of the sections will be considered to have done well. This case tests mental horsepower and the ability to move to conclusions quickly.

Case Interview Question #00586: PROFITABILITY, DIFF COUNTRY, CALCULATING ROIC, WITH DIFFERENT VARIABLE AND FIXED COSTS Our client Murphy Oil Corporation is a major global oil and gas company headquartered in El Dorado, Arkansas, U.S. that owns the whole petroleum value chain: oil rigs, refining, distribution, and retail. Conducting business through various operating subsidiaries, Murphy Oil produces oil and natural gas in murphy oil usa gas stationthe United States, Canada, Malaysia, the United Kingdom, Europe and Republic of the Congo and conducts oil exploration activities worldwide. Our direct contact for this case is the Head of Murphy Oil's global retail operation. His operation consists of two parts: (1) gas sold at the pumps and (2) the convenience stores at the gas station. Recently, the profitability of Murphy Oil's retail operation has declined, and the Head of global retail would like us to help figure out why and to come up with a plan of remedy for the next five years. How would you go about the case?

This case is representative of many of the prepared, McKinsey round 1 cases, in which the interviewer actively walks the candidate through a set of qualitative and quantitative questions. The interviewer should "stick to the script" of case questions. To the effect that the candidate struggles, the interviewer can assist the candidate to get back on track. The candidate should be structured in answering qualitative questions and crunch through any numbers thrown his or her way, always keeping in mind how they tie back to the larger issues. Possible Solution: Question #1: Over the past fifteen years, the overall number of gas stations worldwide has declined by 6%. What do you think might be the causes of this decline? Possible Answers: Consolidation Increase in dollar volume per station Changing population patterns (fewer gas stations in rural areas) More gas stations open 24 hours a day (so fewer stations needed) Set of Questions: comparing gas station profitability It turns out there have been two other changes in the oil & gas retail market. One, the number of gas stations with convenience stores attached has increased. Two, a major new entrant has begun taking market share. Supermarkets have begun opening gas stations in their parking lots. This is not yet a major competitor in the US, where they only have 10% of the market, but supermarkets have 30% of the gas market in the UK, and 60% in France. The next task for the candidate is to understand whether the supermarkets have a better business model than the traditional gas stations in the gas retail market, and if so, why. We'll use the metric of Return on Invested Capital (ROIC): operating profit / invested capital. The numbers for the UK supermarket gas sales are as follows: they sell 10 million litres of gas per year at $1.44 per litre. Their cost is 40 cents/litre. They pay 90 cents/liter in tax. The convenience store's operating profit is $1,000,000 per year. Overhead in the industry is typically 10% of fuel sales, and that's accurate here. The capital cost is $4 million. We'll compare it to one of our typical Murphy Oil gas stations located in downtown London, one of our busiest locations. This location sells 6 million litres per year at $1.50 per litre; its cost and tax per litre are the same as the supermarket. Convenience store profit is 20% lower than the supermarket's. Overhead is still 10% of fuel sales, and capital costs are $8 million. Question #2: What is the ROIC of the supermarket? Possible Answer: Total operating profit = operating profit from gas sales + operating profit from convenience store = [10 million liters * ($1.44 - 0.40 - 0.90) - 10 million liters * $1.44 * 10%] + $1 million = $0.96 million Invested capital = $4 million ROIC = $0.96 / $4 = 24% It also may come out here that the convenience store is responsible for all the profits. Question #3: Without running the numbers, what do you think our client Murphy Oil's gas station's ROIC will be? Why? Possible Answer: Much lower, because of the client's cost of capital $8 million is twice as much as supermarket's capital cost $4 million. Question #4: What do you think causes that high cost of capital? Possible Answer: The location: downtown London. Supermarkets are typically in the suburbs. Question #5: So, given that, what other things that drive ROIC might we be able to affect? Possible Answer: It is likely that we can't change cost or tax, but we could lower price to sell more gas or we could move out of the city. Also, we could attempt to increase convenience store profitability since our convenience store profit is 20% lower than the supermarket's. Client response to our recommendation: The client agrees with our recommendation to focus on the convenience store, and decides to set a pilot program in 1,000 stores. Question #6: What type of new products should the client introduce in their convenience stores? How would you think about what products to introduce? (the candidate was required to come up with eight answers - near the end he was helped along with "Think about what we'd ask if he came to us with product A and product B - what would we ask to be able to decide between those two.") Possible Answer: Some factors we could use to help decide what products to introduce are: What does the existing customer want What products have high margins What can we (and the supplier) support logistically What can we get from existing suppliers What can we link to products that already sell well What products are needed frequently / will drive visits What products are durable What products require little shelf space (space at a premium in these stores). In the end the decision was made to introduce hot and cold food - high-margin, low shelf-space, high-frequency (but low durability). McKinsey is currently implementing the recommendation in Europe.

Case Interview Question #00622: PROFIT, NO TIME TO SET UP Our client Hannaford is a regional supermarket and grocery chain based in Scarborough, Maine, United States. They have about 200 grocery stores as of 2012, and operate in New England region (northeastern corner of the United States consisting of the six states of Maine, New Hampshire, Vermont, hannaford supermarketMassachusetts, Rhode Island, and Connecticut) and upstate New York only. In number of stores operated, Hannaford is now the second largest supermarket chain headquartered in New England, behind only Quincy, Massachusetts-based Stop & Shop Supermarket. Hannaford was doing very well for the last several years. However, from the most recent 2 years Hannaford's market share growth is starting to slow down. Their margins have been falling. Their average sales per store has been decreasing. To make things worse, Walmart recently has opened 4 super centers which contain 75% of the same collection of items sold by Hannaford. And Walmart plans to open even more stores in the region. How do we reverse the decline in Hannaford's average sales per store? How do we successfully compete with Walmart?

Unlike other profitability cases, the interviewer or case giver should not give the candidate time to come up with a framework. Instead pose the first question immediately. Additional Information: (to be given to candidate if requested) Hannaford's supermarket stores are a full-range stores including groceries, pet food, clothing, pharmacy, etc. Some stores include banks, dry cleaners, and post offices. A few others include child care, flower shops and catering. 15% have banks, dry cleaners and post offices 5% have child care, flower shops, and catering The remaining are regular grocery stores. Possible Solution: Interviewer: What are the reasons for the decline of average sales per store? Candidate: It could be several reasons: Targeting the wrong segment Incorrect Location (too many competitors nearby) Inadequate marketing Pricing Inadequate customer service Convenience Owned store vs. franchise - take a look at franchise contracts to ensure that they are motivated to push sales. Economy We have just 5% of stores having all services. Maybe competitors have a 1 stop shop with flower, child care, dry clean, banks, etc. Some stores may be more profitable than the others, but some others are less profitable, thus bringing the average down. Interviewer: We have undertaken a survey among consumers which has showed that the consumers perceive our prices to be 20% greater than that of Wal-Mart. In addition, even though our prices are similar to our nearest competitor Stop & Shop, the customers perceive our prices to be greater than them as well. What actions can the marketing manager take to change the customer's perception of prices? Candidate: Does each shop have the ability to change prices or is it centrally governed for all 200 stores? Interviewer: Each store has the ability to set their own prices. Candidate: Some actions could be Direct to consumer marketing Offer discounts Bring down prices of some products which are significantly higher than our competitor Increase customer services Launch a customer loyalty program Check prices of affiliated services like dry cleaning, child care, flower shops, and catering, etc. Maybe high prices there has created the pricy image for the grocery store Franchise vs. owned stores, check for store managers' incentives to incease price Interviewer: The Hannaford store chain decides that they need to bring down the prices of the fastest selling 6,000 items by 15%. How much would it cost the company given the following additional information? Average sales per store $20 million Fastest 6000 items 24% Next 15000 items 42% Number of stores 200 Super stores 120 Stores over 15000 square feet 108 Candidate: (do the calculations) Average sales per store $20 million Fastest 6000 items 24% Sales of fastest 6000 $20 million * 24% = $4.8 million Impact on profit 15% * $4.8 million = $0.72 million Total impact 200 stores * $0.72 million = $144 million Interviewer: Good. We have the results of our recent market research: the percentage of customers who are satisfied with shopping experience, convenience of store locations, healthy foods, organic meat, good deals, store prices (see Figure 1). The client the CEO of Hannaford is eager to have us analyze the data. Could you let us know the implications of this data for our strategy? Candidate: This market research shows that we are doing well in convenience aspect and healthy foods and organic meat, but we don't have a good shopping experience and our store prices are perceived to be high. Interviewer: Great. Let's assume that the CEO just walked in and would like you to give him the next steps. What would you tell him? Candidate: You came to us with a question of what we should do to successfully compete with Walmart and reverse the decline in our revenues. We looked at the various factors that may affect the average sales/store. We also conducted market research to show where we could improve. Based on the market research, I would recommend the following: Improve the shopping experience Focus on marketing our competitive advantage - convenience of locations, healthy foods and organic meat Work on improving consumers perception of our prices. Interviewer: Thank you. You did a great job.


Kaugnay na mga set ng pag-aaral

key business functions: marketing

View Set

Chapter 25: The Milky Way Galaxy

View Set

CINE 21 Introduction to Film Studies

View Set

ap world chapter 22 and 24 quizlet

View Set

Ricci Chapter 24 - Test Bank - 4th Edition

View Set

LMM NES Subtest 1 Soc Studies and ELA

View Set